Sei sulla pagina 1di 290
FREASONING ABILITY Includes .. questions ‘with __detailed Scanned by CamScanner REASONING ABILITY EXAM G©@ALPOST for Banking Exams Table of Contents Unique Features of the Book ‘ . snk How to Access Videos, é ot. Se aca ult Installed the App? Let's get you started: Ba emcee se Computer Based Test (CBT) Exam Day Strategy... vil Overview of Banking Exams in India Cuarren1: Series Completion .. Practice Questions ‘Answer Key and Explanatory Notes Chaprer 2: Arrangements Practice Questions Answer Key and Explanatory Notes 1 57-78 Cwapter 3: Classification Practice Questions Answer Key and Explanatory Notes 79-112 Cuapter 4: Mathematical Inequalities Practice Questions Answer Key and Explanatory Notes Syllogism .. 113-136 Practice Questions Answer Key and Explanatory Notes Cuapter 137-160 Cwrter 6: Verbal Reasoning Practice Questions Answer Key and Explanatory Notes 161-184 Chapter 7: Coding and Decoding Practice Questions Answer Key and Explanatory Notes Reasoning Ability Exam Goalpst or Banking Exams ‘Scanned by CamScanner Charrer 9: Cnapren 10: Index-QR Codes 0b. wordpréss.cc Practice Questions Answer Key and Explanatory Notes Directions and Positions Practice Questions ‘Answer Key and Explanatory Notes | Machine Input Output .. Practice Questions | Answer Key and Explanatory Notes Data Sufficiency ..... 255-270 | Practice Questions Answer Key and Explanatory Notes | Data Interpretation Practice Questions Answer Key and Explanatory Notes 271-305 Feasoning Ability Exam Goslgas for Banking Exams Scanned by CamScanner ~ 185-204 + 205-226 227-254 255-270 271-305 Reasoning Ability EXAM G@)ALPOST™ for Banking Exe Seprants thorough knowledge of ll the relevant and important topics questions based on the topics. ns is designed to gve banking examination ong with a sufficient number of practice eontains mare than 1200 questions with detaled solutions and explanatory notes for every question Unique Features of the Book 1. Detailed Content Coverage This book contains almost al the important topics of Reasoning. Ability from the perspect Examinations. The entire content of the book has been divided into 12 chapters. Detailed and 1 ive of Banking 1d explanation hhas been provided for each important topic covered in these chapters. 2. Detailed Solution for Every Question with similar types of problems. Cr Te book contains detailed explanatory solutions to each and every question and not ust the answer key. These explanatory solutions not only provide you with an insight into the details of the answers but also help you deal ‘The position of how many digits) in the number 381576 will remain the same after the number Is arranged in the ascending order? (2) One (2) Two (3) Three (4) More than three (6) None of these ET (4) By using the letters T, , S and 0, we can form six meaningful words: STOP, POST, TOPS, POT, POTS and OPTS. (2) OFiginal number form is: 381576 Ascending order form is: 135678 Check the number(s) whose position will remain same in both forms: 381576 135678 You will see that the position of only one number remains unchanged, that is, number 7.Hence, the answer would be one. Crs How many meaningful words can be formed by Using the T, P, S and 0 letters, when one letter is used once in a word? (1) Four (2) Two (3) Three (4) Six (5) None of these Reasoning Abity Exam Goalpstfor Banking Exams ‘Scanned by CamScanner Neston, \/ How to Access Videos This book contains ‘Expert Speak’ videos that you can easily access through an app-on your smartphone or tablet— using the QR codes provided at various locations throughout the book. ‘Step 4 Download and Install the App. To access free videos, you wil need to download the ‘Wiley Test Prep’ App from either Apple's App store (Tunes) oF Google pley store. While instaling the app, you wil be required to type a unique registration code, which you can find on the last page of this book. Ths registration code will work only once and on only one device. type ‘Step 2—Scan the QR codes After successfully installing Wiley Test Prep, perform the following steps to access videos: 1. Open the app and click the ‘Scan QR Code’ option. You can find this option under the top right corner menu. 1e desired QR code and wait for a few seconds till you hear the “tick ‘and found valid, you will be directed to the associated video/ 2. Focus the camera of your device ove ‘sound. Once the QR code has been scanned content. 1. The app login will be valid for 6 months from the date of activation. 2. The app login is valid only for one device. 3, Incase of any dificuly, please reply o the confirmation email that you received (in your inbox or junk email folder) after registering on the app. Installed the App? Let’s get you started: VIDEO LESSON Scan this QR code to watch a video on the exam strategy of the Reasoning Ability section in banking and finance exams and how the unique features of this book will help you prepare for these exams. Reasoning Ability Exam Goalpost or Banking Exams Mee ‘Scanned by CamScanner Chapter 1 Series Completion ‘Scanned by CamScanner Ciera NUMBER SERIES Number series refer to a sequence or patem of numbers following some seqence or pattom. You need to find the missing wrong number inthe series, Different Types of Number Serles There are various types of numbor series, Some commeniy sed number series areas fol: Addition serles: In this type of series, specific numbers based on some pater are acsedto get the next number. For exampe: 2 5,811,14,17,20,7 ‘Solution: Te solution ofthe series eas follows: 5 @ uo om ym 2 TSS WwW Ss ‘The number in place of? should be: 20 + 3 = 23. 2 3,6,14,18,27,,51 Solution: The solution ofthe series Is 2 follows: a 18 gat 72 St OS WT Ww Wr Ww The numberin place of? should be 27 + 11 = 38, ‘Subtraction series: In this type of series, specific numbers based on some pattern are subtracted to get the next umber. For example: 2 50,45, 40, 35,30, 7 Solution: The solution ofthe series is as follows: 50 45 4050? s ‘The number in place of? should be 30 - 5 = 25, 2 42, 30, 20, 12,2,2 ‘Solution: The solution ofthe series sas follows: a2 3 2 2 7 2 Tw es F ‘The number in place of? should be 12 - 6 = 6 ‘Multiplication series: In this type of series, a particular ‘ype of number pattern is multiplied to get the next number. For example: 5,11, 24,2, 53.24, 2, 257.6816 Solution: The solution ofthe series is as follows: 5 11 242 5324 > 2576816 "22 x22 «22 BD v2d ‘Scanned by CamScanner ‘The number in place of 7 shouldbe S324) = 22 = 117.128, vision seria: In this type of series, @ particular type of ‘umber pattern i divided to got the next number. For example 14096, 1024, 256.7, 16, 4 Solution: The solution ofthe series is 2s follows: soos 1028 258 4 The number in place of? should be 256~ 4 = 6 Consecutive even/odd number series: This series is fhased on an even/odd number followed by ts consecutive ‘even/odd numbers end one number is missing in thet series. For example 2 24,6,8,10,7 ‘Solution: 4 is the consecutive even number of 2, 6 fs the consecutive even number of 4, and 60 on. The required number is the consecutive even number of 410, which is 12. So, 12s the required number. 2 3,5,7,2,1413 Solution: 5 isthe consecutive odd number of 3,7 is the consecutive odd number of 5, and so on. The required number isthe consecutive odd number of 7, which is 9.0, 9s the required number. Prime. number series: This series is based on a prime umber folowed by Its consecutive prime numbers and ‘one number is missing in that series. For example, 5.7.44, 13, 17, 19,7 Solution: Given series is the series of prime numbers: Starting from 5. So, the requiced number isthe succeeding prime number of 19, which is 23, ‘Arithmetic series If the difference between a term and its Preceding term is constant throughout the series, then this ‘Series is Known as arithmetic series. In this series, the first term is represented by ‘a’, the common difference by ‘a (aa ~ ax) and Ta (= 2 + (7 ~ 1) d) represents ts n®™ term. For example, 1 4,10, 16,22, 28,2 Solution: This isan increasing series. Here, d =16~10 So, the required term = 28 + 34) 2 17,15,13,2,9,7 Solution: The given series is a decreasing series. Here, d= (15 - 47)= (43 - 15)=-2 So, the required term = 13 +(-2)= 44 Geometric series: Ifthe ratio of any term and its preceding term is constant throughout the series, then this series is 0-4 Shor [4 +(5 x6) = ing Bs Reasoning Ability Exam Goalpost for 89” represented 8 8 fc erm by Ta” Foresare, 16,.8.4.2.17 seta 1=8-10°408"5 at sa tereauredtem=3* 3°3 — ovo: n is ype of eis, 26 © Fires square of particu umber pte Forel ee Suton: The ston of the series 8 oO¥s ‘am we? eh fli] Poa wow Ine ronan pace ot 7s be 17 <9 47 37 wai ymber is @ «instr n tis ope ste, ec run sates eb apacla vont stan ferearot 6.64 ate 2000 / fetdr Te soon othe sere solos: eae ane aoe ! ! | | o ‘The number in place of ? should be 8 => 8x Bx 8 =512. Flbonacc! series: inthis type of serie, the naxt number is w the ation oftwo previous numbers For example F 22, 13,25, 38,7 101, 164 B Solution: The solution ofthe saris 0 follows: f 2 ton tee i ddd z (249) (3125 25136 99489 (634303) The rumberin place of ? shouldbe 25+-38 = 63 n Paranal tere: n tis pe of series, mutile num : Pantems are used aterativelyto frm a sees, UNO For example i 4 2,29.4,25,6.2.8, 17 Sota Te “eMian feaeesi a on: fn fet, yy TT TONS Then me OO Sha 25 94 = — Scanned by CamScanner rf * sotut = oy nes, ae) mown 2s geomet series. 008 S00 at dpress.com 7.43.6.21.7.36.21 | oregon Te SOION OF Ne series 5 on: | Liisi: rent nace? ou ete 12, 8.33.20.7 Tne cotton eet aa, sie ne {ST WS The number in place of ? should be 13 4.4, erator Sere: In tis ype of * Methane anole to get te extranet For example: 1 5,7, 24,55, 2, 215 Solution: The solution ofthe series 35 oeg 5 7? a 8G s The number in place of ? Should be $5 + 8-75 85 +64-2= 117, ARRANGING NUMBERS, In this, you need to rearran, answer the given questions, 2 he 'Be Numbers, as specified, xs For example, + The position of how many digits) inthe number fill femain the same after the number i» the ascending order? (1) 0 (2) Two (3) Three (5) None of these ‘ Ans. (1) Original ‘Number form is: 381576 Ascending Order form is: 135 678 mee bid Number(s) whose position wil remains 381576 t 135678 you wil ee tht the potion ef en one nut remains (eine te monte 7. hence, the answer woud be 2 Wm 37829574, the poston of the fst and elhth ts Iteerchanged aed third and fort Ie nterchanged nthe fn arangement, whlch gt would Be ith rom the sexe? aa ar Be aa as pos (8) ‘To salve this question, you need to velace the number secndng tote nstucton: The orignal numbers 37820874 Now, interchange te numbers on the fest and eign poations ane those onthe thre and our peste: reese rs 37 8 295 7 4 The rearranges number's 47289573 The fh it om the ria ofthe numboris 8 Direction (example 3 to 5): Questions are base on the fve ‘three digh numbers gven below: 203 72 21079536 13. Subtract the lowest number ofthe given series fom tho highest number. Which of the following Is the fst digit ofthe resultant number? wa as as ws @7 as. i) To sole tis question, you need to fist arrange all the numbers inthe sending order, The sequence of aumbers after arranging them In the ascending errs 243 361 472 536679 Subtract the lowest number from the hignest number of thenew sequence, Le. 679 - 243 = 436 Now, the 1 cg of the number 436 is 4 4, What is the sum of the second digit of the smallest ‘eumber andthe thid digit ofthe largest number? chapter 3: Series Completion 203 263 a2 536 619 PES cona wg tne smelt rumberis Trewwsagterbe ugemonoers®. ‘nd ait of he ores be 1 23. . ‘5. Add 4 10 the middle aigt of each number and then ch number. a pangs the fat and tin igs of © ae "curt towest number? ome nears a3 ae vm Bc aperecen on, af «f of a of Sob ke NE ee aL aE Le Ta ae 382. 645986 “he fourth lowest rarmboris = 645 BEE Scan thi QR code to watch our expert speak on number series ALPHABET TEST In the alphabet test, questions are based on the 26 letters of the English alphabet, wbich are as flows: ABCDEFGHIJKLMNOPQRSTUVWXYZ Ouro ese 26 eters, there are fve vowels and 21 constants: Vowels: AE 1,0.U 2. Cendnant 860, 6,4 K, oe ene Sommer 80M AKL MMO RST i OB x gi Te soe the quatre bese on alphabet, ou need to wa Temenbe the poston fies rom iho High ond (rang a he runber inthe scening oer oon The postion aha! tom eo its sons 2[s[-[s[ol-[s[:[o]s]2[a]e] =] | a]s]o] [a] ]a| =] == efefofelrfeful fo lete[ulwlolrlelelststolv[wlalrtz The postion flab om it oe a tous [|] 2] 0|9[ 8] 7]8]8[=[5[2]" [0] °]@]7}eTe] ea ‘Scanned by CamScanner Reasoning Ability Exam Goalpost for Banking Exams ‘Some important points that help you soe questions are 28 follows: 2. Totherigntmeans move fom let tht Le. A107 2. Tothe eft means move rom rit tof. te. 210A 3. From the gt means move fom right tole ie. 210.8 4. From thee means mov from le trig, ie, AtOZ 5. ity find te eter inthe same direction (or example, to the rgniet from certain later counting. fom the rig nen you need to use the subtraction method. Ifyou fe the eter in 9 aterent erection (for example tO the right from certain letter counting from the left then you need to use the action method. 7. Aways remember the concept of EJOTY, which helps you to find the poston of @ letter. EJOTY incites the letter postion in multiples of 5. ep fe 8 fo fs '& Aiways remember the concept of VQLGB, which helps you 6 28 {nd te pon oa ern eevee re WSS ‘indicates the letter position in muttiples of 5. wm ee fe ye 25 Types of Alphabet Test Based Questions ‘Some commen types of questions based on English alphat ‘oe cong based on English alphabet ‘1 Position-based Questions: These types of questions will be based on the postion ofan alphabet, 10] 8 Scan this QR code to watch our expert speak on alphabet test Foresamte, Which letter wit be the letter rom the lt? wa ar 3) na.) The eters arin the same direction. Tus you ‘tO use the subtraction method. auroraeed tes 14 dete a tett=10- Check he 10> let om thee ABCOEFGHEKLMNOPQRSTUVWxyZ The 10% letter rom the lefts J. 4th letter to the left of 14th 20 a 4 Scanned by CamScanner ois i mw set te verter ta tee Sessa Forexomle. 2 sn ae procedas srangoert, Now AY ind followed by a vowel? MUM LIAUDFAES a3 2 £2 tore man £3) none of ese na First, find the letter ‘A’ oe cGy T N@u x LI@u 0 FOES check which “A's are preceded by @ constant end fotowed by vowel: caim@nt nu «Ou 0 BE$ ow, we hve two A that ae preceded By constant Dralehonaty ove! . terete n these questions the series & Mente he gone of bo o% te Teters ins core ater, fou ee to erty #6 patern and Brae end ct the missing teter oF UP Inthe rar enanol Laurea Soliton: The oun ofthe series as follos: ee ee ee ir The leer in pace of? shouldbe 2 letters prio to F 2 AACDE?QIIM ‘Solution: The solution ofthe series is as follows: a Ao De eG yy ™ ¥ as ss Ie ktet nn letter in place of ? shoul eine i Should be 3 the third letter 3B, CD, FF, JH, 2 Setution: The soliton ofthe series is 2 oto: $22 ao ws oo fF H 3 2 aS The parin place of? should beg} ‘cent the pattern fom ana sermons contain a tong arrangers’ "Se types of nae some pattern. You wit nave’! elements Pattern and find o 1 ind out the next element in the arn entity the rangement o™ Ke u Solution: Tho Series starts from the lett ‘A’ and has @ ocuring pattern of 8 few letters. Wnenever, letter’ re “dppears in the series, the last letter le removed from the ecuting pattern of letters an etter "A" again appears in the series. This process continues til you reach the end of the series. Let's break the series 2s follows: ABCOEFS ‘ABCDEF — ABCOE ‘aco ack ana Thus, we can sey that the next letter In the series 8: 5 Arrangement of word In alphabetical onder: In these ‘types of questions, some meaningful words are given and tang you need to arrange them in an alphabetical order. To solve this question, you need to arrange the fist letter of, ‘es ‘each word in alphabetical order, then arrange the second da letter of each word, then arrange the third letter of each ng word, and so on. Ater arranging each letter, you will get he the required arrangement of the word. For example, Which of the following words will come fourth after arranging them in the alphabetical order? (1) IMPROVE (2) INVOKE (3) INMATE (@) INCREASE (6) IMMEDIATE ‘Ans. (3) First find the common letter inthe st postion of the letter of each word, group them and then arrange F words inthe ascending order Rove oK c nee te zeere E ATE a ' 2a sks e r ° ‘Now, find the common letter in the second portion of the letter of each word, group them and then arrange words in the ascending order. ifw@je rove ifwjm eo 1 ATE ifw]v o K€ 1 fey eg tfyjc re a E Now, find the common letter in the third portion of the letter of each word, group them and then arrange words in the ascending order. tm[w]e Di ate iu[P}rove tnfejrease aver | emay goaoees tnivjoxw « “Tne words inthe ascending order ere: IMMEDIATE IMPROVE INCREASE inmate INVOKE “Thus the fourth word is INTIATE 18. Relative Distance between Pairs of Letters: In these ‘ypes of questions, a particular word is gven in which you peed to find out the pair In which the distance between two letters is equal to their distance In the Engish alphabet, Therefore, you need to check the distance of each letter cone by one with other letters. This process continues tit you check the astance of each letter with other letters. For example, ‘a How many pairs of such letters are there in the wor PERFORM each of which has as many letters between ‘thom in the word as Inthe English alphabetical series? (2) One (2) Two @) Tiree (@) More than three (6) None of these ‘Ans. (2) Pick each letter one by one and check the gap between letters in the forward and backward direction as: ‘Thus, we have two pairs. Poe R F NOTE: The process of checking the distance between letters willbe difficult and time consuming when you have along word, ALPHANUMERIC/HYBRID SERIES ‘The arrangement that contains alphabet, ‘symbols is called the hybrid series. numbers, and For example: Study the following arrangement carefully and answer the given questions, Y2TSSZBU%KS9SQAGSQHEMO&SGHTE 1. Ifall the digits are removed from the arrangement, then which will be the 6th element from the right end of the siven arrangement? (ye (2a @aQ @e 6G Ans. (4) Series after removing all digs is: YT$ZBU%KSSQAGQH &GHE ———- ‘The 6" element from the right is E. ‘Scanned by CamScanner Exams Reasoning Ability Exam Goalpost for Banking bots are there In the given 2 Netaeunat cnc of onich i immeitely preceded BY ‘rumba ane food bys et? (3) One WO. ee & tee {3} more na 8) none a V2TsOr BU OKs 9OQAG SOHEM s@sc@re Naw: eck he symbots) whch spreceed bya number anton by ater vr BUSKS Oars Quem 9@xcOre Now, we have thee symtls that are preceded by @ tambon flowed ya eter 3. Fou ofthe taowing fe tons ae ake In 2 certain ‘ay based on thar postions inthe shove arrangement and s0 form « group. Which le the one that does mot fellow the group? (2) 258 om (3) 908 (a) 50e (5) 986 Ce PRACTICE Ty +4 Directions (Q. 2. following series? 1A. BAB, CBABC, DCBABCO, EDCBABCOE, 7 (2) Feveacoer (2) FeDaABDEF (3) FeoceABcoEr (4) GeDceAacoeG (5) GeDcacoES 2. HER, JGT, LV, NKW, 2 (a) Pay (2) omy (3) Px () Pax (5) ow 3. 2Y,UT, PO, Ku (a) be (2e0 (3) GF (a) FE (5) None of these 4. Figg, EeFt, Dats, 7, Boce (2) cae (2) Aace (3) Deco (4) coda (5) Coos 5. GH, KL, 2, Tu (4) mnior (2) Nopg (3) oPoR (4) PORS. (5) None of hese & ZOwWU, rxwU2, 2, Wwuzrx, vurKw (a) xwuzy (2) xwwuye (3) xwuvzy (3) xwuzy (6) wuz peer Scanned by CamScanner Ans. (8) hos a diferent patter to get the Net letter i, S50 has @ group. ‘hoen: Sean this QR code to watch our exper, speak on alphabet series a @ 2 as 1 7 we 1 os 8. KP, KLQ, LMR, MNS, 2 (y hor (3) PNS Py Lad (5) None of ese % 2820 840 210 42 7 9 3° 2 o87 ©) 1a7 13 410. Find the alphabets conoid coriases "486 he een sen) S00. set_ese_sos_t (2) seste 11 How ‘many symbole) teary rangement each th and preceded bya congetieh Is mean, D WenaeRrea ier, (3) Tree (8) Na, 18) None of hese than, Wee 35, 17. 19. a 42. on the baste Gar out of five are ake. Which of the following does ‘not belong to that group? (ner ae (3) KO (a) Pr ur Direction (Q. 13-46): What will come In place of °' In the following series? 43,5 7 40 15 22.7 46 ™ ‘tering Gy 26 (2) 29 A0e9| at @)33 wy 637 ue 44, EFGHU, EFHUG, 7, EFIGHI,EFGHU Pen () etic (2) EFGH (3) ERICH (a) erucH (5) erucn SY 5 068 42 52 8 7 an (7 (79 (82 (3) 85 ‘87 46. 1200 600 200 50 7 167 @a 2) 125 (3) 10 (@ 115 jn 47. How many pair(s) of letters Is) .re there in the word 'ARRANGED’ each of which has as many letters ‘between them as there are In the English alphabet? (2) One (2) Two ) Three () More than thee (6) None 18. How many pairs of letters are there in the word STURMERIC’ each of which has as many alphabets, (forward and backward direction) between them as there are in the English alphabet? (2) One (2) Two (3) Three (4) More than three (6) None of these 49, What should come next inthe following letter series? JJUIMJTHGJIHGFJINGFEJIHGFEDJINGE ED as Qc @B ai (5) None of these 20. Which of the following will come in place of the 11 sere "question mark (2) in the following series based on the 122, Which lottor wil bo the 7th letter to te right of 42m later om tho tetin th Engh alphabet? ar Or QR aa os 23, The position of how many dligi(s) In the number 15314687 will remain the same after the number I arranged inthe ascending order? (4) One (2) Two (3) Three (@) More than three (5) None of ese 24. DW,HS, 7, PK, TG ay Lo 2) iM (@) MP am (NP 25, What term should come next in the following letter series? GHIJKLGHISKGHIJGH ws QH @ as ok 26. Four ofthe following five options are alike in a certain way and belong to a group. Which ofthe follo.!ng does not belong to the group? yar (2) 23 19 at 627 27. 58132074461 (a) 29 (2)30 @) 31 (4) 32 6) 37 28. 800 400 200100507 (4) 20 (2)30 25 (a) 35 (6) None of these 29, Which letter will be the 40th letter to the right of 12th {etter from the let in the English alphabet? ay QT @u aw Ss 30. How many such ‘t's are there in the given arrangement each of which is immediately preceded by a ‘D' and followed by a vowel? AHRODKEIDSKWTUDKOEF DK FBDSW English alphabetical order? —(1BPS Clerk MAINS 2015) (2) One (2) wo aca FE ? (9) Three (@) More than Three ay (2) sw (5) None oftrese am (a uz $i cere wat o (a) NML Num belt 21. The position of how many digit(s) in the number (3) LN (4) MUP. ont" 243698 wil remain the same afte the number le (5) NPL arranged inthe ascending one ne 32. The position of how many digitis) in the number (3) Three (4) More than three crargad lh femain the same after the number is OMe wranged in the descending order? see (1) One ne) To (3) Thee (4) More than three (5) None ofthese a= . Se ‘Scanned by CamScanner king Exams Reasoning Abily Exam Goalpos for Banking 45, 22, 23,27, 36, 52, 77,2 a's sgt and 218 init 2) 60> 3%, Suppo 1 aed to each odd paces Gh umber 6 313 Oi ibaa om each, own ple (5) 4 ‘tzsnoss. ich of th : inte tart place Hom he et Difficulty Level: MODERATE ae 1-8) What wi co 2 Directions (2 men as Peg or er following series? 2) nthe number Tei ie ne er in om? 78364834 wil remain the same after the numb a 2K feet go oh ax - Bees (8) Are than tee (5) UK (5) none ofthese 2 XG,UF,0),L02 s (2) Fu 2 35. YW, VU, RR, 2, G1 qa aM (2) NM (3) EV an (3) MN 4mm (5) None of these ene 3. AV, ER, 10, MM, 2 38. Which of the otowing wil come in place of the (yee RL question mark? (3) PM (4) = Baw Use Pam 2 Foe ew ity Bim (3) JG (4) Kin 4. 4,484, 49284, 2, 493817284 Nx (i) 4937360 37. What tom shou (2) 4937204 senesg™ *2Uld come next in th folowing teter fal agrees ie TSRoPoNMurs (a) 4936064 ue TsRaronrsnaprse & assrze¢ an om a 8. BcoHUNOPTUy? 98 428-040 E56» 52) ay a2 ) Glow i ; (3) 1124. 8 ae (8) None of these. (5) 1164 © 426,447,390, 326,2 | Directions (Q, 39-45): What ¢ ‘Question mare inte tte came in place of the (3) 199 99. 4,6,9,14,20 a5, wettest 5) 208, aia 2 Bat ae (i Joy 22 49 89 a69 9 oe ase 2) 297 ~ aes. 13, 729, 1391, 9 (5) 323 (4) 317 = 2197 gas (2) 1864 AW, oR, a, 2, eae Sse BF any 1 a ay 33 36.20,2 Sr (4) TF Ox 2) 26 8 22 39 95 (9) None ofthese War (201 $8 63 01 eq 18, 42. 1680, }) 254 (2) 203 $40,420, 7,105, 525 (8) 265 (4) 257 (3) 260 (2) 365 10. mow £5) None of 4) 210 2) unp SY OS sq 43. 5.2874, 143, (3) vor (2) » 4,143, 2 ui thats (8) np (8) Uae 13 (3) 245 22s 1 00 up 235 6 200 ) 23 ace KY ey 9,20, 63, » 3) Res (2) 90, (2) 256. (5) a 3) 26 2) 259 = (4) Sor (5) Mone of thes (4) 249 10 = ‘Scanned by CamScanner a yoursmahboob. wordpress. GQ sie cmvie.om Jon (32-14): Sty the following. Infomation ett shows ha ventions gen bee Sintttres saseonevoneasneny 42. How many numbers) t/are there Inthe stove rangement ach of whichis mela followed by % MIgrert procmdoa by 6 consonant? i i reo (3) twee (3) more an tree (5) None o ese 13, Wilh ofthe flowing I the 5 tothe gt ofthe * trom the lft ondn the above arangonents te oe ar oe oe 14, 1 ll te numbers and symbols ae deleted from the ‘hove arangement, then whlch af he folomiag wil be 3 tom ne et ond? ae (a as ay ok 15. How many pairs of letters are there in the word "MANPOWER' each of which has as many letters (forward and backward direction) between them as there are in the English alphabet order? (2) One (2) Two () Three (4) More than three: (6) None of these 16. How many digits are there in the number 789452 each ‘of which Is as far away from the beginning In the ‘umber as when the digits of a number arranged in a descending order? (2) One (2) Two ) Three (4) More than three (6) None of these 17. How many pairs of letters are there in the word ‘TENTATIVE’ each of which has as many letters (forward ‘and backward direction) between them as there are in the English alphabet? (2) One (2) Two (3) Three (4) More than three (5) None of these 18. How many digits are there in the number 84796135 ‘whose positions remain the same after arranging digits, ina descending order? (2) One (3) Three (5) None of these 19. In the word ‘MANUFACTURER’, If the positions of the first and seventh letters, similarly, the positions of Second and eighth letters are interchanged and so on, which letter is tenth from the left end in the new (2) Two (4) More than three arrangement? (Va Q)N au ‘at (6) None of these 20, How many pairs of letters are there in the word [GEOGRAPHY each of which has as many alphabets (forward and backward aection) between them as there are in the Engish alphabet? (2) One’ (2) Two (3) Tree (@) More (5) None of nese Which letter Is seventh from the right end if the frst half of the word "MULTIMEDIA Is written in a reversed ‘order and the remaining second half are also witten in 21 the reversed or ae am @I au (5) None of these Directions (Q. 22-26): Each of the questions Is based on the following three-digit numbers: _(IBPS CLERK MAINS 2015) 624 923 488 843252 22. I all digits of each of the numbers are arranged in a cscending eer (within the number sol) and then {he numbers oe avanged in an asconding order, which will be the middie digit ofthe third number? a3 (2) 4 @2 ae 8 ls subtracted trom the mide dig af each of the 7 umbors and then the postons ofthe fst and third Sigs are Interchange, whieh ofthe folowing wl be the tat cgi othe second highest number ao fey trearanged na descending order? a9 (2)7 (2 @a 4 24. What will be the difference between the last digits of the highest number and the lowest number after the positions of the first two digits in each number are interchanged? (4 7 @2 aa 63 25. Which of the following is the sum of the middle digit of the lowest number and the last digit of the highest ‘umber? aa 7 @2 as 63 26. If the first digit of each number Is replaced with its bredecessor and then the numbers are arranged in a descending order, which of the following will be the ‘Second digit of the second number from the top? aa Q7 2 (48 6) 3 27. Which letter will be the 5* letter to the right of 11% letter from the left in the reverse order of the English alphabet? (Qt 29 @)4 ar ©) k W Scanned by CamScanner me se 7a 8 ke vic ef te fotoing i te sand dt of the 2 aad by mbercing te Honest nambe fomtengtmne? 7 ae a2 er 28. tn teh of tee munbor, the sum fal the toe Gites own mae? areas (2 752 aa (i aie 5269 230, Ad 4 to te fat eg of wach nomber and then Inwrchnge te et and hits ofeach rumba. Vion oh Yaloing wil be te tid ght of the second highest number? ae ae a6 @s er ‘34 What i the sum of the second and tind digit of the second iowest mmber? ipa an 335 a (9)17 pee ‘32 Subtract trom the third dig of each number and then Interchange the frst and third digits of each number, Wich of the folowing wil be the frst digit ofthe thied ari bs a2 33. 112 9 subtracted trom each odd dg, and 4s spelen dg in amber 7652346, ee ie to a a 8 om the rg and td tom as ‘an 614 me ie a aang in a at tm cma reverse order it Sorel Somers cra Peecaae tae ‘Scanned by CamScanner wordpress.com |, aK oe meta of the following wor ‘conus 37. wnletang thom in the alphabetical order? ic 2) MEASURE Meow (3) nwo = 3) nacre =] following words will come last ot the 28 ie mtn te sphabetial order? fame ta) vewoicr & vanier (a valley tS veR0N . ft et 20 ltrs are given I the roverse order * the remaining In the same order, which letter witt re carat le of 5th eter rom the oe ae (aN (ak aH “a we Direction (Q, 40-44): These questions are based on the three-dlgt numbers given below: 7 #63 que 756 «282 BAL 40. If the frst and third digits of each numb interchanged, then which of following will be Towest number in the new sequence? 379 (1) 148 (2) 292 (3) 973 (4) 354 (6) 657 50 41. M148 added to the first digit and second digit of ea ‘number and then the numbers are rearranged in ascending order, find the third digit of the seconé highest numb a3 25 @7 os as 59 sa ‘Second highest number? (aa (a) 237 (3) 535 (2) 418 (5) 379 (4) 567 44. at abe te from th Mi thir aig of imercan te HS 0 hn gf ge28 ren it the third highest number in ‘ (2) 282 yoursmahboob. wordpress,.G.QUs comson (4) More than Trwee (2) ve "70 ay, 8) None ection (2 48-48% Four of tho following fve options are anette Contain way based on the Englsh siphabetca! se, tnd so frm a group. WAich ls the one that doce nat fotow the erour? Matty 46. 4) Toge yw (3) Ken (a) wer (5) ano ‘rm, ang (3) LOK 2) wav mie 3) RUD (@) ore " eur +. (yay (2) ¢sMo (3) su (@) Freo © fy (5) MOK Directions (Q. 49-50; What will come In place of the ‘question mark (2)In the following number series? ‘re 49, 50000 10000 2500 500 1257625 the (a) 55 2) 25 @) 60 (a) 335, 6) 31.25, 50, 7, 13,25, 49,2 (dy 9 (2) 97 ‘ach (@) 89 (a) 87 the (6) None of these ‘ond Directions (Q. 61-55): The following questions are based on the fve three digit numbers given below: sol 462 «318675924 ‘51. fall the numbers are arranged In the descending order he {rom loft to right, which of the following will be the (a) 22086 (2) 27434 difference between the numbers that are second from (3) 28644 (4) 29564 the lft and third from the right? (5) 30788 (a) 144 (2) 462 (@) 129 (4) 357 5.342 78 186 61 7? (5) 84 a od (2) 132.8 + we wentstnwnenitactonn mane Bit ae ‘many numbers thus formed will be divisible by three? () Ove ‘2) Two 6. LAX, N6V, P8S, 7, 7484, v240 (3) Tiree (4) Four (4) R120 (2) R150 (6) None (@) Ria (4) R160 (6) R130 ‘53. If all the digits of every number are arranged in the ‘ascending order within the number, which of the 7 DTG FQK, HNO, JKS, 7 following. will form the highest number In the new (2) wu uv arrangement of sumbers? ) Law (4) LW (1) 591 (2) 462 (5) JLV a on (4) 675 8. 5 30 185 1300 ? 93650 (2) 10400 (2) 4040s 54. What wil be the resultant If the fest digit ofthe lowest (3) 10505 (4) 10550 umber s muted with the rd dg ofthe second (6) 10600 ighest number? ren on 9.6 22 86 342 1366 7 @)20 Bes (a) 4832 (2) 5264 (8) 24 (8) 5452, (4) 5064 (5) 5678 _ 55. If the positions of the first and third digits of each ‘number are interchanged, how many even numbers will be formea? () None (2) one (3) Two (4) Three (6) Four Difficulty Level: HIGH Ltt each vowel In the word ‘MOTIVATE’ Is replaced with the next letter 23 wall as each consonant Is replaced tnith the previous letter according to the English phabet ordor and the new group of letters are frranged Inthe order. Thon which ofthe following letter wll be fifth from the right end? we (20 @k we (6) None of these If each vowel In the word ‘TEMPTATION’ Is replaced intth the next letter as well as each consonant Is Toplaced with the previous letter according to the English alphabet ordor and the new group of letters are ‘ive order. Then, which of the following arranged Ietter will be seventh from the teft end? ae @0 @s as (5) None of these ‘3. If the letters in the second half of the English alphabet are reversed, then which letter should be the 7% letter to the right ofthe 6 letter from the left? yx Qy @N aM Oz Directions (Q. 4-15): What will come in place of “?" In the ven series? 4.1.2 33 200 2373 7 43 ee Scanned by CamScanner 19. How many such numbers are Reasoning Abilty Exam Goalpos for Banking Exams 0 om * oe 2) 780 (3) 785 are some ae. ses ii 0 oie eat i a2 a i793 99 ad am gee Ee BRS 24. 008, ane ani Bie rene sh a7 26 ss on tim} fase "dso ame aie ams Direction (Q. 16-20% These questions are based on the {oltowing letters, numbers and symbols arrangement. Study ‘the arangement carefuly and answer th questions below: SSRSGANBO7UINISEIARNT AT 16. Which ofthe folowing wil be the tir tothe lef ofthe thirteenth fom the left end in the gven arrangement? a7 ae ‘3s (a3 (5) None of these 37. Mf all the symbols are deleted from the given ‘arrangement, then which of | the foloning letter/number willbe elghth from the right? an ae Qa a7 (5) None of these 18. How many such letters are there inthe sven arrangement each of whichis Immediately followed by ‘a number and preceded by a symbo!? (1) One (2) Two (3) Tree (4) More than three (5) None of these there in the given mangement each of whichis immediately fellowed by ‘symbol and preceded by a vowel? {2) One 2) Two (3) Three (4) Moe tan tee (5) None of these J ei 20. nich of the following shoud replace the question 14 arent the kiven series based on the aren ‘arrangement? San v2 a 7 24 f (2) 2s ae (4) na (5) None ofthese “Scanned by CamScanner Directions (@- FY 1 a 2. 28, ee 22.28) Sige lana MAINS 2035) at SRT OIF EUK Whee senisitas toe, ebro How mame fowal are tere. in the Gy tree (3) Fe {6} None of nese (2) Four (@) More than fve fe deleted from the following will bering) How many letters are there between the etter that im trom the left end and the letter that is 15% from fient end? (2) One 2) Two ue (@) More than three (5) Nene of nese Which ofthe following Is 3° to the right of 22% ftom the left end inthe given arrangement? wa 2s aH a6 OK Wall the vowels are changed to the next letters in English alphabetical order, then how many Fs will therein the new arrangement? (1) Three (2) Four ) Fie (4) Six (5) More than sx What should come next in the following sequence? 9783186427531642 ws @7 Qs (4)3 4 ow many sch alts of lters are there inthe “CREATIVE each of which has a0 many ters renee, thom in the word in bot Tormand ana yee Sirens) a inthe alphabet? oe 1) Two oat re than (5) None of these ene 2% 308 428 7 544 ee ae ‘pas (5) 49.4 nes 05,058,065,09,7 o8 (2) 0.82 (5) None of these. oss 6, 12,30 305 e725 > sesstors (3 ants ta were (5) None of these (9) 2545.28 yoursmahboob.wordpress:eomr= ttn ter i forwarded by two 4a, ase 7-22 feters to. ge the net term ¥ 22+ 14-33 “Therefore, the required term 14 . (#2)(6 + 2)0W + 2.8, PMY. $a aa+t 3. (4) Te sees in the fist eter of At Hence, the reaured term is 33. fact term 73 series, the next 2) In the given series, 2a Ua gh te age digit M4 en found by moving the third ae, re ler from the left of current term The sre tested eter of alt at the endo the term, ach terms ; * a2 : Hence, the term in place of i eontleentneenteea Hence, tne equted term is EFUGH. Hence, the required term in place . or FE 8. (4) In the given sores, letters In 28, (3) The pattern ofthe series is 4. (6) n he even series, each term tach term consists of their 36+7=43 contains four laters in which the predecessor. a frst lotr i coptal flowed by Hence, the equied tem is (M + the same smal letter andthe tid B)N44)(8 1.0. NOT. 534 13-66 letter i the successive letter of frst flowed ty te some letter in ®._(3) The patter ofthe series is 65+ 16=82 sma The sere ne tes of mn¥s200 mexsor ind ris ie 840 +4=210 Hence, the required term is 82. « ef] ja)-aya 210+5=42 46, (3) Me pattem between the terms { Geis ofthe gven series is: o¥? Fe 1200+ 2 600 - faf-aj-a Hence, we can say thatthe term in 600» 3=200 yay place of? 1. 200 + 4= 50 “yf pal-ay a (2) In the given series, the word 50+5=10 ‘eset’ is wren four consecutive CUE Y times. Thus, the series. is 10+6=167 at ala) 2 ‘sesetsesetsesetseset. Hence, the Hence, the term in ss alphabets that fll" the blank So ees tiie y spaces are ‘seste’ 47. (2) According to the gen word aig 1 (1) The symbols) which is/are : ; Hence, the term in place of is "AR ae oi a Tigaemiotay, wake ‘ARRANGED, the pal of letters « preceded by @ consonant: cs (2)In the given series, the number ARRANGED oflewers wincrecseatyene inte, CEB CArearaussorevsncaseery in the next consecutive term. You Hence, there is only one symbol in t can see that the leters in each the above arrangement which is From above we can see that there term ofthe series are consecutive immediately followed and isoniy one such pair eters. You wil get the fist leter preceded by 8 Conguromt of the next term by two letters 1B. (2) According to the given word ahead of the lat letter of the “TURMERIC the pairs of letters is: ee Scanned by CamScanner TURMERIC Hence, we can say that there are ‘wo such pats. 419, (2) The gwen series starts with J Each time one backward) letter {according to the Engi alphabet 's added in the. previous term, Hence, the letter'C comes nest in the series, 20. (4) Taking the fst letter of each Par, the series is ARSC Hy ib osesy Taking the second letter of each Dir, the series is Ath B 56.28 5).17 0.0052 24. (2) Original number form is: 21.3 698 Ascending order form i: 12.368 9 Check the numbers) whose Position will remain the same ater ‘Comparing bots the forms. 2 pee 2238 6 You can see that the position of only two numbers remeing lunchanged, ie. numbers 3 and 6 Hence, the answer would be Two, 22. (5) The letters are in ditferent rections. Thus, you need to use j the addition method. Rigt= 7 stert=12 Lefts 19 ‘Now check the 19m letter from the lef. ABCDEFGHIJKLM NOPQRSTUVWXYZ The 496 etter trom the lefts s, (21 Original number form is: § 3.4 4687 Ascending order form is: 13.456 78 Check the number(s) whose Position will remain same ater ‘Comparing both the forms: 18 ‘Scanned by CamScanner “Reasoning Abilty Exam Goalpost for Banking Exams vba d avai tenet em ne nee Set is er 33 sree sae 74 2 a ar The pair in place of ? should be 10, 25. (3) The series starts from the letter ‘6 along with the recurring pattem of few letters. Whenever, letter ‘6 reappears in the series, the last letter is removed from the ‘recurring pattern of letters and letter G' again appears in the Series, Tis process continues til YoU reach the end ofthe series Let's break the series a follows: GHUKL GHisK GH GH Thus, we can say t letter nthe series i 26. (5) 17, 23, 19 and 41 are all Prime “cumbers, while 27 js diisible by 3 and 9, tthe next Hence, number 27 does not belong tothe group. 27. (3) The solution ofthe series is 08, follows: BB 1320 > 44 6 meth 7 The number in place of ? should be: 20411 31, 28. (3) The solution ofthe seties isos follows: 800 400 200 100 50 > 242 122 an The number in place of ? should be:50+2= 25, 29. (2) The letters are in different rections. Thus, you need to use the adaition method. ight = 10 eft = 42 tett= 22 ; Now, check the 22 letter the let ABCDEE KL ROP ORST UMW XT? ‘The 22% letter from the left i , 130, (2) Maret letter": AnROBEIDSBWT UoBoEF@rsosw Check Ks which s preceded by y and followed by a vowel AHRO@ELDSBWT uo®OEFO@r ssw Now, we have two KS which ara preceded by ‘D and followed by vowel 5 WS 45 yt Pep lee? so i LUNN, 32 (4) Original number form is: 8 3 7912 Descending order form is: 9 8 7 3 21 Check the number(s) whose Position will remain same’ after ‘Comparing both the forms: S32 tw a 9 ‘ 10 hat esto creo Pst tarsi et THe Se: place's digit 8nd subtract st oo each even, Place’s qi 2 trong Now 16 naw"yiey aee i Chapter 1: Series Completion. Tre four defom theleRinine 38, (4) 6 -4=2.9- 6-3, 14-9 _pitfieulty Level: MODERATE ‘i ew sequence s 3 =8.22-14=Band53-22=13 4. (4) he sere in the fst leer of om au py odmalucier tenis 789 Since the diference between each a om rs banat term of the seres folows Fbon Jy | eo te sex ™ order form is: 213.94, sos cere, the ciferonoe series inthe second letter of ve pemcgeriactimietss4e; FA Me esis mamta Toe ses ine 45 21 13+ 8). Hence, tne required s Bet ‘ehiay, ‘check tne number(s) whose term=35 +21 =56 wee nten santo postion wil remain same’ after ‘The required tm is XL 2 au 40. (4) The terms ofthe gven series emppactng bein ey are 3%, 53, 73, 93 and 14. By —_2._(2) The series in the first letter of rapes aaa folowing the same pattern, the con term ® taba it # we ‘eaqred term is 13» 2197 ded by “p a XApU4-Qaoloek You can see that the positon of 44 (5) The terms of the gen series é oe en ee follow tne pater as: The series inthe second fetter § Unchanged, Le. number 3. Hence 3142013 eaenrermis the answer woul be One. taeoese Cody Ftp tp 0-85 U ‘hich are 35.3) on Theretore the required tem in lowed by oo a 4 16+ 4=20 wre ery CYvyYY So\ the recited numtor 3. (4) The series in the frst letter of w go om FY oy | stg M \ ONE NSS 42. (4) This is 9 decreasing series. Aah Eat | eM ate ‘The pair in place of ? should be Each team is half of its previous And, the series in the second t MN. term letter of each term is a 36. (4) So, the required term is half of hancimeaumrairaae Sos eg 420, Le. 210. Therefore, the required term in vould be af Ne YY De 4% (4) The pattern followed by the place of is QL. terms ofthe seres: 4. (©) The pattern of the given series “8 The pa in place of? shoul be ae : arn place of ? shou - 2873 IH. 28+ 4674 2x2=4 37. (2) The series stars from the 74+ 69-143 22*22= 484 letter ‘along with the recurring w 222 222= 49284 wh 443 +92= hose patie of few lets. Whenevee $+ 925288 letter “T’ re-appears in the series, 44. (1) The terms of the given series 2222 = 2222 = 4937284 the last two letters are removed follow the pattern as: 22222 « 22222 = 493817284 2 from the recurring pattem of eens letters and the letter “T” again Therefore, the required term in ‘appears in the series. Ths process (9*2)+2=20 lace is 2222} ie, 4997284, continues til Cesena nn end ot (20%3)+3 5. (4) In the given series, three nof tn ‘Consecutive letters are writen; the ains Let's break the series as folows: Sesean nes ext three consecutive letters are ce TeRQroWML T8HO5G (3x4) +4=256 lef. the next three consecutive WN IsRoP” Tee 45. (3) The pattern ofthe given series letters are written and so on, a {5 as follows: Hence, the term in place of 7s Z. Thus, we can say that the next ws iawn Gesu 2241-23 6. (4) The pattern ofthe series is: a 4) 23+22=27 425 -28=417 * oa on 27+32=36 417 - 38 =390 CY YY + 390 - 48 = 32 FY sense ve \ 85+ 52=77 326 -59=201 a T+6= 201-6 =-15 The pair in place of ? should be enn 16H, -15 - 72-358 1. = —_— Scanned by CamScanner 19 king Exams 1 Goalpost for Ban ‘essoning Abity Ba | Hence that in place’ 7s the series ofeach term o 7 (8) The patern ofthe series is 1 yt ty) get are two such pars S+5033 her wean say ratte in ARMENIA umber 4) 33+ (6*2)=23 “ft 84796135, pace ot? OCR 230 0%2)=43 42. (2) The numbers) wich 8/0 Number ater arranging. in 43620» 2)=63 83+ (40*2)=163 103 + (80 2)= 323 an preceded bya consonant Hence the requied term is 323, © (A) Te series inte fist eter of cach emis ‘and preceded by a consonant AoC 4 mss ys ey Pre Fry The series in the second letter of from the left end means ach tem is fourteenth from the et ond WER ema gy sc Now, fa the leer, symbol or The required term is SH umber tht is fourteenth rom 3 (8) The pattern of the series that the ett ence prove i SUN OFS HO wearers !Gorevincassere Hence the poten: Hence, we can say thot 8 is iby ibeey fiteenth rom the teen age 4A. (5) er deteting the umes and tape ‘Smbols the new arrangement Seraares WRERTABSOKY 384 63=401 HEReTy 63+ 101 164 ence, Eis 5® from the rent end 1iKigh-aey inthe new arrangement The required term is 265, 10. (2) The pa first tetter tem followed by the Of each term of the 48: (3) Te folowing Sure shove the required pai of eters inthe eheg Word "MANPOWER. The pattern tolowed Tro I ‘eter of ech eof te ceneens, t tl} iat a, Hence there are tree sich pr The Pattern flowed by thd eer texters 48. (2) Te orgnl numbers 73005 Wt pralal after arranging in descending oder is 9874 Hence, the required term is uPo, IL (2) The pat The boston of 8 and 2 remain tern followed by first unchanged, as leteroeacntemmortheseee . 47. 2) Aezording tothe even worg (tee TENTATNE: the pas often The batter ‘otowed by secong leer eschiemmorthe vse 20 ene ‘Scanned by CamScanner ie eter tern flowed by thi a The pat Immediate folowed by 2 vowel WeDo caragBeeesneaseerr Hence that tere are 2, we can ay numbers, which are immediately folowed by @ vowel (5) Fito to the rent of the ninth TENTAT ve From above, we can See that descending order's 98765434 en st tng eae 20. (3) According to the elven. word ‘GEOGRAPHY’, the pairs of letters Cy GEOGRAPHY tu Hence, we can see that there are ‘ree such pairs. (4) After arranging the according to the given cons the new The 7 teter from u ew arrangement sl, 23, rere of ae yoursmahboob.wordpress.com ‘Now, on reversing the positions of the fest and third Gigi, the rnumbors become: 415 319 874 338 242 Arronging these numbers in a escending order: 874 415 338 319 242 We get the second highest number to be 415 whose frst ig 4. 24, (4) After interchanging the first two digits in each number, the numbers become: 254 293 848 483 522 From above, the highest number is 848 Tne lowest number is 254 ‘The last dig of the highest umber is 8 ‘The last digit ofthe lowest number is4 So, the required diference = 8 - 4 =4 25. (4) The lowest number is 252 ‘The highest number is 923 The middle digt of the lowest umber is 5 The last digit of the highest umber is 3 So, the required sum =5 +3= 8 26. (1) After changing the first digit of each number to its immediate predecessor, the numbers become: 424 823 388 743 152 ‘Now, arranging these numbers in a descending order: 823 743 424 388 152 We get the second number from the top to be 743 whose second digitis 4, 27. (5) The letters are in the reverse order. Thus, you need to use the ‘addition method. Right = 11 tleft= 5 . 28, Now check the 160 letter trom the ler MiggsiHGreocea 416" letter from the left So, the 16th letter fom the lefts * (4) To sove this question, you ‘need to first arrange all the ‘numbers inthe ascending order. 289 476 538 753 a4 ‘Subtract the lowest number from ‘the highest number ina_new ‘Sequence, Le. 814-289 = 525, ‘The 2" digt of the number 525 82. (4) First, you need to find the sum fall three aigts of each number, 476-4 44766" 17 538-+ 5134816 289-4 2+8+9= 19 814+ 8r104~13 153-9 74543 15 ‘The sum of all three digts of the umber 538 is 16, which is an ‘even number. (3) Add 4 in the middle digit of ‘each number: 476 538 289 B14 753, | el “f wl sf 576 638 389 914 B53 Interchange the first and third digit of each number individually: yy y KRKKK Now, rearrange the number in the ascending order: 358 419 675 836 983 Now, the 3° digit of the second highest number= 3 digit of 836 is=6 ‘Chapter 1: Series Completion 34 (A) To solve this question, you need. to frst arrange all the umber in the ascenaing order ‘The sequence of numbers in the ascending order i: 229 476538 753 aa a mene a Sansui Saeaminrs scarica mec seeenitit a oe eens ciuoucas so 5% 439493 saad i eieorans Sane eam az KER Mm ON Now, rearrange the number in the ascending order: 287 318574 735 82 The fst gt ofthe tid highest numbers agt of 574-5 33. (3) To sove ths question, you feed to int subtract 2 rm each 60 places gh and add 1 each ven places at tldadaa The new sequence ofthe number is 5731359, The second digit from the right in the new sequence is 5. ‘The third digit from the left in the ‘new sequence is 3, ‘The sum of second and third digits is: 543-8, 2 Son Scanned by CamScanner king Exams Reasoning Abilty 1am Goalpost for Banking vestion, yOu 194, (2) To soWe this a need 10 repace the number dccorang othe instruction: The exgnal numbers 4579927586. the third Interchange the its at ‘2nd ith positons, seventh. and hin postions, ane eighth and {enh postions: need 4896728675 The thid aigt in the rearanged umber. The sith gt in the resranged number is 2 The sum of third and sith cigs is: seed 35. (2) The series after reversing the First 13 letters so folios: MLK JING FEOCBANOPQR stuvwxy2 The letters are in the same lection. Thus, you need to use the subtraction method lett =15 Left= 9 NNow check the 7® letter from the left Menai me OcBANoPOR Stuvwxyz Thus, the 7 eter om the et is 36. (3) The series ater reversing the fist 18 letersi a follows: ROPONMLKJIHGFE OCBASTUVWKYZ The letters are in the diferent iets. Thus, you need to use the edition method, tet =10 +Rienes 9 Rights i9 Now, check the 19 letter from the right. Scanned by CamScanner ROPONMLAIING EoCeASTUVUXY? Thus, the 19% ter from we Cah bs oi te oe mes wae ee ae be dscinig ore Tec wien s mila GA ° " was Ac Ei Now te common eter inthe 2 poaton of te eters each wor goup. them and. then Brange "the “wares in te ascending order. MP]G azine Mials ori ry Mialo Hiowe MTElc HANI Cc Mfelasure ‘Now. find the common letter inthe 2 postion of the letters of each word, group them and. then, arrange the words inthe ascending order MaAfe]H 1 ow MaAlela 21 MalTjo wt METS uR M efe]H ANIC The words in the ascending order are: MACHINE MAGAZINE MalonTy MEASURE MECHANIC Thus, the second word is MAGAZINE, iat find the commen leer n 3 ir pecton of me tetany och word, UP em and tae, arrange the words nthe sscending order. VALUE vier Dic + vjAR IT ET ¥ viALLeY vier ston ymmon letter inthe Now, find the co ition of the letters of each so gale them and. then Wrange tre words. ithe ‘ascenting order. v[A]L UE vija{R to eT y¥ viaje Ley vie}R Dc viel; so Now, find the common letter in the 35 position of the letters of each word, goup them and then ‘arrange the words in the ascending order. va v E Le le 1 A A ETR]o elris 1 Gears N Now, find thee 4° postion of word, “group arrange the ascending oder Vaur Yau ‘ommon letter in the the letters of each them and then ds in the Ts, the lst words VERSION. r ‘99, (2) The series after reversing the {iret 20 letters as olows: TSRQPONMLKJING FEDCBAUVWXYZ ' Ihe ites are in the same . Gc. Tru, yu noed to we be ‘tesutacion mabe. ven wets z ae ie ow check th 7 ter rom te tee Greocekuvwxyz Thus, the 72 letter from the left IN. 40, (2) To solve this question, you eed to first interchange the frst e fre third gt of eoch number i indo a awe a wy x x Now, rearange the number in the ascending rr aa e255 e778 Now, the lowest_number in the ‘new sequence is 148, 41. (5) Add 1 to the fist diet and second dig of each number: Hn ‘Now, rearrange the number in the ascending order: 392 489 663 866 951 Now, the 3% digit of the second highest number= 3+ digit of 866 is=6 42. (2) Arrange all the numbers in the ascending order. 282 379553 756 841 a “Tne frst digt of the. second highest number is. 1 dg of 758 7 The third dgt of the second highest number ie. 3° igh of 7566 ‘Te sum of the first digt ond third ligt ofthe second highest number i761 43, (4) Rearrange the cigts of each umber. Number Ascending Order 379379 756 S67 282 228 san 18. 55355. The sequence of the number Series in the ascending order is 148 228 355, 379 567 ‘The highest number i: 567 44, (3) Subtract from the third gig Goma oy ag 289 o4p 8 ap yp aap ape arb rat doh seb ash Interchange the fist and third aigt of each number individually: KAKAK Rearrange the numbers in the ascending order: 048 182 255 457 873 ‘The third highest numberis 255, (3) Mark the letter‘ Fim@owrns ce@c@® AVOoeOungisze Check ‘P's which is preceded by a consonant and followed by a vowel: Fim@ow ene eOc@a vO@ve@ungiszr Now, we have three ‘P's which are preceded by a consonant and followed by a vowel. YIU has a diferent pattern to get the nex eter in the group. a7. (4) ayy Fc hos a rent ptr 0 ot the nox eter nthe ou. oo x—242—8.1 ey Beaten —2o0 vey hee pte See eo w— 2s oe) He \VxSU has a aferent pattern to get the next iter inthe group. 49, (2) Me soldion ofthe series is follows: PSTN The number in place of ? should be 125 * 5 28, 50, ()7+6=13 13+(6*2)=25 25+(12*2)=49 The required number = 49 +(24x2)=97 52. (6) First, rearrange the number in the descending order: 924 675591. 462 318 Now check the number second from the left and third from the right: ‘Scanned by CamScanner 23 02 318 The umber is 675.591 = 84 {52 (5) Fst. add one tothe ti apt of each number Now, the new amangement ofthe umber series i ga 43319 Crs ‘Check which numbers completely vse oy 3. 592+ 3197.33 463 +3 154.33 3193» 106.33 676+ 3=225.33 925330833 hus, we can say that no number ‘S completely dsb by 2, 5.) Rearrange the cigts of eacn ‘umber in the ascending ores Number Ascending Order 591 te5 42 ang 318439 675567 924 2g fe tavence of reeranged ‘numbers inthe ascending order 338 195 24g 24967 The highest number i 567, The original number is 675, §4 ) Arange the numbers in tne ascending order: B18 462504 675g 5 NSEC ofthe nest umber 'S I diet of 31863 The third ig of the second Gus number is: 30 gga ed 675=5 ——__ Scanned by CamScanner ing Bams Reasoning Ability Exam Goalpost for Banking pict aot of te fest Se mest nomter and 8 moarnber Sette ond aos swe he ft Te rota mmber ndaaual: 7 number after interchanging teapute sis 1208 oe its ruber a ae 0 en Inetes 26 on 378 Difficulty Level: HIGH 1. () The new group of letters after ‘replacing each vowel with the next letter as well as each consonant withthe previous letter accoraing to the Engish alpnabet order is swuasr ‘After arranging eters according to the English alphabet order's BFILPSSU The ft letter trom the right end Of the new arrangements ®FiMessy —— 2 (2) The new goup of ltrs aftr [enlaeng each vowel withthe nee later 28 well @s each consonant rena tevous leer accords Sr egSTeiSh alphabet order SFLOsesspy tne ATaNEI etterssccorcng to BRS alohabet onder o BFILMOPSss reg eventh eter tom te tet endof the new arrangement fF tM omss, Sara 3 2) The series of letter ater crepe t second hat of Cor English alphabet i. tne a the right of eign om treet o4 Means feet fo the left ena scot eM KMD EW Vergy teri te ot | the lefts ¥. (2) Te potern ofthe series aeatate2 23433 3345+ 5%= 200 200%7+73=2373 2873 x9 +93= 22086 Hence, the required term 22086. 5. (6) The pattern ofthe series 34(04%3) = 3412-42 42+(12*3) > 42436 18 78 +(36*3) 37.8 +108 186 186 + 108 « 3) => 186 + 29 =51 814 (924%3) => st+972 48.2 Hence, the required term is 148: © (2) The series in the first letter each termis SoMa yr a, The Series in the second letter each term is The series in the thira letter 00h term Xaaviays a Oa ay The “eaulted term is R139, 2 | % (3) Te serag ‘ in the fr Sach term jg N° First letter doe 2a ee: Pere oot er wd e each the secong letter off yay re term oa Ly Be sere in ai Saehtomis” third eter of 4 OAs keg — even, Saw] gy tas iy ™ Series ig. 2 6 ! 12 15 _—__,... yoursmahboob.wordpress.com p46 +54 185 3s +7+5= 1300 1300# 8 +5 = 10405 40405 + 9 +5 = 93650 ‘Te require term ts 10405. ‘8. (3) he pater of the series is (6+4)-2022 (22% 4)- 286 . (96 » 4)- 2-942 (242+ 4)- 2° 1966 (1366 « 4) - 2= 5462 ‘Hence, the required term is 5462. ag * 40, (2) Te patern followed by the terms ofthe given series is: 3306641" 547 547 +5941=673, 8, 224 673 +441=738 673 +3941" 766 766 +29+1=775 The required termis 775. (4) The pattern of the gwen series is . 2eDereat 41+ 342=40 40+4963=107 107 +53 +4 = 236 236+ 6 +5 =457 The required term is 236. 42. (4) The sens in the first letter of each term is Att ty tt ‘The series in the second letter of each terms Voter stu tty The Series in the third letter of ‘each term is. ° bat use Hence, the required term is Mi4M, 13. (5) The pattern of the series is, 12+(4+2)41=15 15+ (4+3)¢1217.33 ——__ 17384(4+ 4) 4119.33 19.33 +(45)¢442113 Hence, the required tem is 21.13. 4A, (4) The pattern in the fst letter of each terms Ka, Wa, ua, Rag The pattem in the second letter of ach term's WA, US, Ab, Kee ‘The patter in the third letter of each term is VA, U2, RS, M26 F Hence, the term in place of 7 KM. 15. (2) The patter ofthe series i: 224307 74344925 2544+5~105 305 +5 +6=532 531*6+7=3193 3193 « 7 +8 = 22369 Hence, the required term 22369. 46. (2) Thicd to the left of the thirteenth from the left end means tenth from the lft end. ‘Therefore, from the above, we can see that 7 is tenth from the left end. 417. (2) After deleting the symbols from the given arrangement, the new arrangement is: 9R5AHB7U2JS63AN2M 41 Therefore, from above, we can see that 6 is eighth from the right end. 18. (2) The letters that are immediately followed by a number and preceded by a symbol are: SEB eee rue ser aWe 19. (1) From the figure below, we can ‘see that there is only one number a Scanned by CamScanner Chapter 1: Series Completion nich is immediately followed by © Symbol and precedes by 8 owe: sseseaveor@Dpiseveenzucet 20. (3) The pattern followed by the first letter of each term is: se ee ‘The pattern followed by second letter of each term is: ‘Te pattern followed by third leter cof each term is Hence, the required term is %2M. ‘2. (4) The consonants that are Immediately followed by vowels ‘Tus, there are six consonants Immediately followed by vowels. 22. (5) After deleting the vowels, the new arrangement is: FYFPTCISKIGI FKTHP ‘Thus, K is ninth from the left end Jn the new arrangement. 23, (3) Let's find the letter that is 8 from the left end and the letter that is 15% from the rignt end: Thus, there are 3 letters (C, A and 4} between 0 and S. 24, (4) 3 to the right of 12" letter from the left end means 15° from the left end. Now, let's find the letter that is 15 from the left end: Thus, G is the 15% letter from the leftend, 25. (3) The new arrangement after replacing the vowels with the next letters in the English alphabetical order is: FYJVFPTPCBIS KIGIFFVK PS T HPF 25 ene sees tos 0 Secret GeeGerrcrsreG Beer Thm re ae §F 9 P Trteonen coset Thre ae ees % Omacrorogt so 08 Dit Series Even Dg Series 97531 3642 7531 a2 531 a2 an 24 ‘According tothe sles, the next umber 65. 2, (3) Pck each etter one by one ‘and eek te gap between eters In btn the forward and backward irecsons whose dstance is equal to the gap between them in the Engish alphabet ‘Thus, wave tree pas, 28. (2) The soto of the series as falows: 36 388 428 7 546 62 AnnaKnKwW 28 +4 452 464 476 W242 a2 via The number in pace of ? should besa28+52=48 28. (3) The solution ofthe series is as folows: 05 055 065 08 > +05 34 45 42 +05 405 405 The number in place of ? sh beos+o2=1 rd 90. (3) Te so fol: on of he Serles is as ‘Scanned by CamScanner ahboob.wordpress.com scning ty Ba oxpat for Ba ams vn pace of 7 20 Re aags .5~ 2098.75 ne ees 88 aa. eye souvent fotows: 7 3 98 392 78 29 Waa ap 758 e442 224 Ce! a ‘ne number in pace of ? srould per29 0224514 ‘32. (2) Te solution of the series 98 faows: 49 19 7 79 159319 43710 +20 +40 +80 1160 aoa ee ‘The number in place of ? should be: 19+ 20= 39, 133. (4) Pik each letter one by one ‘and check the gap between letters in both the forward and backward directions whose distance is equal to the gap between them in the English alphabet “Thus, we hve four pais. 34. (9) Makin symbol cuszo@sKi2P04 W@xe 3 Us@AveR© 7H1@3 90y Check symbols) which is preceded by a) number and foloned tyaleter: cM5z0@) PSF tovstina a na ate preceded by a number and followed bya letter. L2P04 veR® 38. (9) Sees aor emoving al letters 5&24%agg #95 6 or. ‘Now, check the nm the rights: 582 to. *38S Gory element from sazsnsegooriee ‘thus, the 8® element from, signs ferent pattem to Ho2 has a ait ‘he next letter in the BTOUD. a pt Wi ice 7 sill 6F8. 38, (4) The letters are in the sar direction. Thus, you need to the subtraction method, Rieht= 22 —s, Now check the 15 letter from: ‘The 15% leter from the rigntis (3) Pick each letter one by ‘and check the gap between in forward and backward di Thus, we have thre pats, 40. (125+ 46~174 4714 46 %2)= 268 263 + (46 x3) = 401 401 + (46 x4) «585 585 +46 x5) =515 41. (2) The pattern of ft isasfolons, \ e*" (20%2)-203 @ yoursmahboob.wordpressont™ ‘The first series = 454, 648, 642, Sy (on~2)-2°74 43, (4) The terms of the series follow " the patter 05: 3036 ty qa) 2= 346 aaa Inthe ft series, each number is \ (aus = 2)- 2290 fe {Jaa more. than the previous (290 «2)-2°578 1200 + $09 number. ‘a2. (2) The pattern of the series Is as The second series = $27. $24 on mens oe fate vensees ech nce xoyetn43 Inthe second series, each numt oe {307 more than its previous (3 *5)+2=67 296 + $02 = 999.2 number except 733. (67443-2721 ° 45. (3) The gen series ison 1 3)+4=817 400 (reveasing series in which Bch (27a 3) 890.2+ 75 7 800.84 term is twice of the previous The required term = (817 * 2)+5 frumber except 30. It should be 32 mess 44, (6) The given series is formed by in place of 30. ‘combining two series. Scanned by CamScanner Chapter 2 Arrangements ‘Scanned by CamScanner ’ Seaeieee aia Toons.» £0 eal ie ‘Bing on the basis of comparison or postions of other persons, geass wncice, stm ctr or ae ot bee eiencmn scene a nionred peel ere Wien veiem sot cassie ace aae Gaerne tees row and rank is always from top bottom of the row. In banking exams, diferent types of questions are asked. Some ‘of them are shown with the help of examples: viDEo. ‘Scan this QR code to watch our expert speak on ranking ‘Type 4: In this type of question, the positions/ranks of a person from both sides in a row are known and we have to find the total number of persons in that row or vice versa, ‘Total numberof persons = (sum of pastons of same person trom | bath sis... let nd rip side/top and bottom) ~ 3} Example 1: in a row of students, position of Mithoo from left, ‘ide of the row is 35th and from right side of the row is 26th. Find total numberof students in the ow. Solution: Total no. of students = (Position of Mithoo from left + Position of Mithoo from right) ~ 1 =Total no. of students = (35 + 26) - 1 =61- 1= 60 Position ea person trom opposite side = (Tota no. of persons - Postion of same person trom gwen sie) +3] Example 2: In @ row of 20 persons, position of Ankit from right ‘ide of the row is 15th. Find his position from left side of the. ton? ‘Solution: Position of Ankit from left side = ((Total number of ‘Bersons in the row ~ Position of Ankit from right side) + 1] ‘Position of A trom left side = (20 - 15) +4 +1=60 Type 2: When the positions of two persons are given from Opposite ends and we know the total number of persons, then wwe have to determine the nuthiber of persons between these ‘wo persons. In this type two cases arise: Th sum of psltons.of he te parsons rom ic sess tan ne ea umber of persone ewe no overlapping = nthe et ion ample 3: ow of 54 persons is 35h om Srttrcw ana 820 onthe re ie othe vom. Fn the rovofpersns siting omtween Aare? Soon: Hore Sum of postions of && 8 fom opps ends Sas PSO Total no, of persons + No. of persons between A & B = (Position of A from left + Position of 8 from right) - Total no. of students ~ 2 = No. of persons between A & B = (35+22) - 54 - 2= 57 - 54-2=4 Case (lit: If positions of different persons from any side are Biven and total no. of students is to be calculated then it ‘always a case of ‘cannot be determined! or “data inadequat fr ‘can't say. AS in this case we do not know if there is overlappit.g or not Example 5: In a row Position of A from left side of the row is 18th and position of 8 from right side of the row is 25th. Find the total no. of students in the row? Solution: Cannot be determined as position of different persons is given from the same side, ‘Type 3: When the positions of two persons are given and their Positions are interchanged, after interchanging position of 1st Berson is given from same side as before interchanging, then (i) position of 2nd person from the same side as before interchanging = Position of 2nd person from same side before interchanging + (Position of ‘1st person after interchanging - position of Ast person befose interchanging from same side) (il) for finding the total no. of students, find the person whose Position from both sides can be depicted from the Oe ‘Scanned by CamScanner anking cas Ping iy amon ig nope xen ten 8 oh ios 20 von! ‘tem = Difference in n cee rrerson whose postion from (0 te rare of are te hay a same ae tetrad ater merchangn BNE eae hay ae tang 2m of eefbeegsintnyitiar ‘the row and Vaithay iS Sencar craton 2 mrranisnasaineies © neem laa it seer ett er ccc aeren ier fne on es = ecg ESnmesen emt tr (34-24) - (Vaionav’s postion trom right "postion from lef before = Total umber of persons = (99 + 34)- 4-73-1272 Sun of Pens between Shika & Vaibhav = (sion of iene tom left ater inerchanng- Postion of china yo left before inerchenging) <1 Tagg OY opetens been Sina & Vahey = (34 24) - i=t0-199 cppase te Pons of wo dierent persons ae gen rom mea este Om a thd pene esting the vo ¥9 2 is poston i gen fom etfs heya efit fom ether of the wo persone bt porns nett 20d We have ocala he washer Persons in the row. donee wees Of Westen fst fod the poston of the ge pane 2 8 of the ow and then find te eo persons accorcing to type 1 thet 7.8900 of persons, poston o tom et Side of Sma & Poston of fom apn sac ie is tr csthest in mide oa gta pg ‘Of R fom Jat ste of the row is 160, then tnd narra umber of persons inthe rom: Sen: Poiton of rom eft 160 nd Pom lets 10% Sra te 30 5 ( 16 - 10-3) persone ny siting between P anos Sse Ris stg in mee or Pane there must ‘0 be 5 persons stig between Q ana 1B the poston of om ret = Posi off SEE he poce "EN = Poston of @ from right a ‘Scanned by CamScanner jons of Fro 1s = (Sum of posi of persons “otal suber we ereett+25)- eo yo ucets, pon of Heena fom in a row of gra for ig 3de of theo Exam © wis 12th and ME rade of Heeno & Megha end sovorte in sting vt Hm now many students are aaa froena ath then y in fe ovation: Postion of Sahl om te of Heena Position jon of Sahil fo Solution of Sani from Heena = 32+ 8 = 20th P t eena and Sani i siting in mile i tn fom Hes “Sahil is at th poston Rol then a t-31-1=30 Ww? intnero en a fon bed oe Megha Position of Megha from right rom right = Position Pasion of Sanit *oraen RStrctomivonpe a0 8=200" at number of stdents (Sum of postin of a set} i ‘= Total no, of students = (20 + 28) - 1= 48-1 = 47 ston where we have {0 calcu the mma ruber of persons ina row ten Ri oa cose of orrapo tions of persons from ei Casa of cvroping ie. en post = rom oh Sides overlap each ober, then minimum number of persone Sum psttors of persons om Bath sie Perso between them = 2 Exampie 8: postion of Deepak from left side of a row is 16th {nd positon of Shavesh from right side of a row is 18th ang only 2 persons are siting in middle of Deepak & Bhavesh, Fir the minimum number of persons that can be seated in iis Seating arrangement questions, PUZZLE it into an sors. 50 hat, the congas” ca answer the SwestONS asked nthe examinoae’ Sita. ute appears in ng aenattns Sth one cecton, Ye, 2 set of Understand the Conclusion in siong You can, ) To solve th aR yA © Buz pr, 4 trent tPetnd the pes Yeu negg P then prepare the truth gee tabje and insvewerng the question hel, _- ‘you can easly Ne the questions with the help of @ truth {pole. An arrangement of elements provided inthe row-column armot's called a truth table. In this arrangement one element ‘Sahown in arow and other element in colum, Let's understand the puzzle problem withthe help of examples ‘vibEo ‘Scan this QR code to watch our expert Direction (Q. 1-8 Study the following information carefully _and answer the questions given below. P,Q. R.S, and T are five persons in a buliding. Each of them plays one game namely Football, Cricket, Hockey, Tennis and Boxing, not necessarily in the same order. |. Pand R both neither play Cricket nor Football 4. Q4and T both neither play Tennis nor Hockey. fi, Sand T both do not like to play Football and Tennis, |v. Preither plays Boxing or Tennis. ¥. S.does not like to play Cricket Solution: ‘This puzzle can be solved withthe help ofa truth table. First, prepare the matrix in which rows represent the persons ‘and coluras represent the games. From the point (), mark (x) under the Cricket and Footbal! {games column in front of person P and R. From the point (i), mark (%) under the Tennis and Hockey ‘games column in front of person Q and T, From the point (i, mark (x) under the Football and Tennis ames column in front of person $ and T. After using first three statements, we can see that only one box ‘is unerossed in column Football and infront of person Q. So we can say that the person Q plays Football. Now, mark (7) in the ‘box and mark (*) all row because one person plays only one game, Now the truth table is as follows: Ge) ee | Cae) ee P @ R * * s x ™ T * * * From the point (iv), mark (x) under the Boxing and Tennis ‘column in front of person P, ‘ter using the fourth statement, we can see that, only one box unerossed infront of P and under the column Hockey. So we Scanned by CamScanner yoursmahboob.wordpress.com ‘Now, mark (7) in the can say thatthe person P pays Hockey. Now, mark (7) inthe ton and mark (2) in the remaining blank boxes Hockey. Now the truth table is 26 follows ne box is uncrossed in ‘After markings, we can see that, only one box is ‘column Tennis and in front of person R. So, we can say that the i (¥) in this Box and mark (%) person R plays Tennis. Now, mar all boxes in that row. Now the truth table is as follows: ee an @ ¥ 7 A * * R * ¥ * * * s i a x T * * * From the point (v), mark (x) under the cricket column in front of Person S. After using fifth statement, we can see that, only one box is Unerossed in front of S and under the column Boxing. So we ‘can say that the person S is into Boxing. Now, mark (¥) in the box and mark (x) in the remaining blank boxes in column Boxing. Now the truth table is as follows: Q R * ¥ x * x s * * = v x T * * * * ‘Now, we see that, only one box is left in the truth table. This box 's related to the column cricket and in front of person T. Now, mark (¥) in the box, because all the remaining boxes in column ‘and row are marked with (x), 3 nking Bas ingAbity Exam Goapost for Banking Reason Now moth abe as ftows: ut not necessonty inthe some: We oc seven one No foor below Ft Nori a fe ot odd numbered Noor Oot hevwacant Noor i not OF ‘even numbered floor. g the vocart eso an ere tune beoween Et round oor S268 Ta Wve med Povo eto onthe ith fo to: statement, A aerdea tl tt dors vcart and the vacant fore it for tear bln ‘ter preparing uth table, the ei Persons and games sa ows fons. between 2 Wilh of he toning came payed by sp (2) Footbon (2) Hockey (3) Tennis (4) Cricket (5) Bou ns (6) Ths ater anabsing Wich ofthe fotowing Persons plays cricket? 9 2s ae at oR 405 (8) Th ater anohsing tuts abe, we 20 say thatthe / Person T plays Crcket 4 [nl ote towing came payed by Qn (2) Tennis (2) Cricket (3) Boxing () Hockey (5) Footbai 4. (5) Thus, after ana ng truth tabe, i * Me Can say thatthe Person Q pays Footbal "INE POrsons plays Tennis ae ar aR as 69 ‘rs (3) Ths ater anabsing uth tabi We-can say that the Person R lays Tenn Dhrsction (6-10) Study the sronine nfornaton caretuy a peseate the gutstions gen pont Abuling has seven fear abated from 4107, in such » 2.8 20UNd for tat ramos 2 fist foors numbered 2nd $0 00. One each of he 8008 A, BC OE: and 34 ‘Scanned by CamScanner ruth table, we can say thatthe Person Ss into the Bong numbered for We find the following figure: - ‘aca fer not 08 number) the |g Ames Ls f-2 0 the even statement, Ory D lives between Eang Eomornnaatienaats We find the fotowing Figure: 6 + vacant oor 5 Ales cies ru ALA. id fe bas Ly nor te he Ben statement 8 doesnot live on the, flor. E does not ine immecin We ind the folowing gure: Broun "ly below oF above the foor of os 8 = vee, oe 4m 2a 2 te Now, questions (6-10) hos follows: — We nso te toon wa 0 we Ee 1 (8 Tm wo can yy ottommost floor i 17. Whe Hives between the 6th and the 4th floors? aoe “or 20 as ga Be Be sos. (3) Thi. we CBN 3ay at person A ves between © and aor ree, 8. On which of the following floors does B tive? ren ais (2) 20 Ba we am ‘ans. (5) Thus, we can ay that person B ives onthe 7 floor . 18, Which of the following floors Is vacant? aa 238 (a) 7 ae os ‘ans, (8) Thus, we can say that 4 Noor is vacant. 410. Who among the following lives on the odd numbered floors? (2) 80.6.4 acer Q@GADE MARES Eanae ()AFD.C ‘ns. (1) Thus, we can say thatthe persons who live on the oda ‘numbered flor are, , A.D, and SITTING ARRANGEMENT Siting arrangement is @ process of making a group of people ‘it 9s per a prefixed manner. These questions are based on a set of information including a certain set of conditions. The following gure illustrates some common sitting arrangements: pes me 1 a et is Either 1 or 2 sets of questions are asked from this section. In each set, generally 5 questions are asked, meaning overal 5 to 10 questions are asked from this section. Thus, this @ very important topic to obtain good marks in the examination. Some general tips to resolve questions based on sitting arrangements are listed as folows: 1. Identity the type of question. 2. Keep in mind the directions which ae given, 3. Decode the information given in question with the help of ‘symbols and notations. 4. Start your arrangement from that object/person/thing which appeared most of times while decoding the information. 5. Analyse such type of information which is hidden in nature or ‘elated to cases of possibilities and prepare the respective cases. 6 At tne Concepts of Directions Chapter 2: Arrangements stage, set the remaining objects /tnines/ ‘emsons that re sil penaing to arrangement Tre Concept of “who” In siting arrangement, the word 'who' refers to the person oF thing that is before ‘who Example of who": There are 8 persons A, B, C, D, EF, G and H siting around o circular tabe facing towards centre. Bis 31d to the left of E who is 2nd to the right of H e ‘The Concept of “and” In siting arrangement the word ‘and’ refers to the person or ‘thing that lies atthe starting ofthe sentence. Example of “and”: ‘There are 8 persons A, B, C, D, E, F, G and H sitting around a Circular table facing towards centre. B is 3rd to the left of E and 's 2nd tothe right of H. rs Scanned by CamScanner A tein th concept of whe” “orate deceing tegen a iar table. Sereno 8... Uand Tare sng ound acu Liretecheeetmwen tase Ame ASOCE OY asata] {me persons ae sting ooposte crecton othe centre. Vis Zn to tne le of Us Indo the nent of A.B is rt the eM F.C dnd to the right oT Ats an mmediate neighbour * a TSS fe to watch our expert ‘Scan this QR code to wat speak on sitting arrangement ‘Anew te letowngqvetion by selecting he mest appropiate oton. Drmcuny Fy Level: Low an (2) 102 3) 16 (a) 168 6) 192 5, Whats the resultant value ofthe given row? 42 46 7 488 3 (tan 088 number (3) 490 (2) 890 Netoneniwn dey Necearcnen An (6) 2670 we s aoe (5) None ofthese na cenTUne! & folowed ty the sncter even mete te adieg Sum. ttnhe ubery © Maal ranked Sth from the top and 46th from th bottom In a class. How many sti clase? te 04 rumber sto re there in the. owed by the even number, even onthe asa oe Acute alowed to rome en Note: The operations are. ‘@p@eR On Number trom tet to right. ° vende notte. One a ez ee HOW many gis are thee (2) 102 a (5) None of the eas 2 zh val ty LAS mete tt ns carl a Serta ‘spective ranks from thin th top. What ‘emutant abe othe second 8 batto zi nalise (2) 26 ana 29» a 2 os 3) 27» ana aon 2) 25> angsqe 3 as (5) Cannot be Getermineg ) None ofthese 3) None of hse 8. Navin b 14 ran 3369 4: 2 ao ‘Scanned by CamScanner 16. a7, Direc ans Sib hot Bhaw Rajen yoursmahboob. wordpress.CQM, , srarsenens “44. whlch le the fifth number to the left of the number which is exactly Inthe midale of the following Sequence of mumbors? 98765432198765419876542219876 a ae 2s (309) (a7 5 12. ina class of 49 students, Madan ranked 23% from the bottom, find his rank trom the top. 236 (2) 25" (3) 30 (8) 29" (5) 27" Directions (Q. 13-47}: Read the following Information to ‘answer the glven questions. M,N. 0.P. Q. Rand S ae playing cards siting around 3 forcular table facing towards ‘the centre, but not rnecessaniyin the same order 1, isnot the neighbour of O and Q IML Mis the neighbour of Nand 0, |W. S, whos second tothe left of P, i the neighbour of Qanak, 413. Which ofthe following s correct? (2) Nis between Mand P, (2) Pis between R ands, (3) Qs tothe immediate right of S. (4) Ris to the immediate let of S. (6) None of these 14. Which of the following will be R's position after Interchanging the positions of @ and R? (4) Immediate neignbour of Sand 0 (2) To the immediate right of P (3) Immediate neighbour of § and P (4) To the immediate left of M (5) None of these 415. if Qand P interchange thelr positions, Q will be (1) immediate neighbour of $ and 0 (2) immediate neighbour of R and N (3) between 0 and M (4) to the immediate lef of 0 (5) to he immediate ight of S 16. Which of the following shows the group of persons sitting in a sequence? (a) PMN, (2) NOM (3) SRP (4) Sog (5) Qo 417. Which of the following is correct? (1) Mis to the immediate right of N. (2) Nis tothe immediate lef of P. (3) Qi between S and N, (4) Sis between Q and R. (5) None of these Directions (Q. 18-22}: Read the following Information to ‘answer the given questions. Six boys are sitting on a bench facing towards north to be Photographed. Sanjay is to the left of Rajendra and rignt of Bhavesh. Mahesh is to the right of Rajendra. Rohit is between Rajendra and Mahesh. Sandesh is sixths from the left. — a 22. 26. 27. Who ts iting immediate right of Rohe? (2) Bhavesn (2) Rajendra (3) Mahesh (4) Sanjay (5) None of these Who is tied from the let end ofthe row? (2) Bhavesn (2) Rajendra (3) Ront (4) Sonioy (5) None of these Who Is cocond from the right end? 2) Saniy (2) Rajendra (3) ron {@) Bnavesh (5) Manesh ‘What Is the position of Rohit? (2) Second tothe left of Saniay {2} Immediate right of Mahesh {@) Fourth from the right end of the row (2) Thed from the eft end of the row (5) None of these nis second tothe ign of Rohit? Or Saney (a non (3 enn (8) Sandesh (3) none of hese na group of 35 stunt, Rajosh gat the 9 place in the can” sooma gt the Te place after Rash What Soema's place wom the bottom? TIBPS Clee Main, 2025) (ay a7 (2) 19% (3) 20" (4) 220 (5) 25" 4m a row, Mohan is 25° from the left and Shyam is 9% from the right. When they Interchange thelr positions, ‘Mohan becomes 27% from the left end. Find out the ‘new position of Shyam from the right end of the row. {SBI PO Mains, 2015) (a) 24 (2) 22 (3) 23 (4) 24 (6) 25 ‘Some students are standing Ina row. A student, Sohan, |s ranked 25° from left side of the row and 19 from the right side of the row. What is the total number of students in the row? (SBI PO Mains, 2015) (a) 26 (2) 39) (3) 43 (4) 48 5) 49 {In a class of 42 children, Joseph's rank is sixteenth from the top. Kevin Is seven ranks below Joseph, What. {s Kevin's rank from the bottom? (S81 PO, 2014) (4) 22nd (2) 20th, (3) 29th (4) 23a (6) 25th {n a queue, Arun is 16th from the front and Jay is 29th from the end, while Mahesh Is in between Arun and Jay {f Arun be ahead of Jay and there be 56 persons in the Queue, how many persons are there between Arun and Mahesh? as (29 (3) 44 (4) 10 (6) 12 37 rg Scanned by CamScanner 1 Banking Ecos Reasoning Abilty fam Goalpostfor Banking bottom 28, Muka ranked 450 frm top and 28th om th bet snvecas How may students are hero inthe ies (2) 28 13 aa wa Directions (Q. 29-23 Read the flowing Information and namer the question gen below. Ina cu, en members & 8,0, E, FG and H ae siting around a etal abe. Out of ei, fur stat comers of 8 {able ana fae outside, nile ote four ati the mda ofthe soe of ie als and foe towards centre of he table Dats tr tothe rt of Eats ted to the let of. faces the cere. Chi one person sts betwoon Ear FF not the immediate neghtout of CH faces te cenire. Ais not an endate negnour ot F.C dors nt stn the mie ofthe £29, What tho poston of A with respect to C7 (2) Fourntorhetet (2) Secondo to et (3) Fh othe nae {2 St saga oppose to coh cer (5) Secondo te re 30. Four ofthe folowing te ing fhe are alo ina certain way and se fom ene eeu heh ene ote ftowing ons nt ao a ao as en a represent the mombers @)ienar ne? ©) Gann (4) Danae Whe among the me fotmin stro the net ot go oe we Wich of the totowing arconenty SOMME ante he sain Immediate ign of As (2) 4 GandD face oui (9) FH and aces cones Ann 008 not ike Ant ad ata a it CO. The SP thes that oder ant g 8S Due 24 Yellon, though mee ne a, Ou an SL The £0 8,2 OGP. The person wo hens eeessarily in ot necessary 4 Me DGP ke wtite and may renee COUT is aI the same order antag COU though is not an Sk Amit a DM. tke red, nu noe one who likes white colour is an aso moms _—— 2 immer tern et an Pein, B Sesoes gan oc ecnmnmgoortenet eee Be tio, Crag tenn iet Sate ren Ba neous Whois C0? eS (2) amit @) ara (a) Aaa {5} Over than te given options Which colour is hed by Ani? {1 Yelow (2) Black @ Res (3) Pink 6) Bue Dietons (@. 39-435 Read the following informa areal and answer tho given questions. fait nen, Amar Aiea, Aan, Ata, Ait ond Anta siting around a cular table facing towerds the conte, ‘not necessarily in the same order. : wed ‘Ankit is second tothe right of Amita, Ami eibour of Amita. Ankita Is second to tne een ‘Amina i third tothe right of Ankit. Aman is secenee tc, on ‘Amita and third to the right of Ameen, sere Se ey 8 hosts tothe inmate right of Amita? fens (2) anita (5) None of these nee, Ai st between, (2 dona" — lina, Aman maaan (8) None of these. ea aot How many (starting trom Amer, 2 1) None" 3) Two (5) Six 0 es pink cok Colour, is a DGP. Anni, ae ta Wh ‘Scanned by CamScanner Pend ty So! >Bangy Directions (0. 44-45}: Read the following information rep and anewer the even question Inarow of students, Waseem is 12 and Yogendra Is 20" tom ttetetona i 44 How many students are sting between Waseem and renee Yoosndre?™ (oo as (3) Esther (4) 0 (2) (4) Cannot be determined (8) Other than the given options 45. How many students are in the row? (24 (2) 20 (3) Eitner (1) or (25 (4) Cannot be determined (6) Other than the given options Directions (Q. 46-50} Read the following. Information carefully and answer the given questions, Mira, Vieioo, Vii, VineSh Vinod, Vikas and Way ae siting around 9 circular table facing towards the centre, but ee necessarily in the same order, Nintoo is fourth to the left of Vijay. Vly is to the immediate {lehtof Vinod. Vikas is third to the right of Vinod. Vipin toute {o the right of Vinesh. Vineshis not an immediate neighboor of way ae es St 46, Whos siting In front of Vijay? va wrasen (3 veh y (3 vc weit cameo dezmzes (5) Other tan the given option 47. Who sits between Vinesh and Vinod? (2) None (2) Vikram (3) Viay (4) Vintoo (5) None of these ‘48. How many person(s) ls/are siting between Vinesh and Vinod (starting from Vinesh in clockwise direction)? (2) None (2) One (3) Two (4) Tree ©) Four Who is third to the left of Vinod? (2) Vikas (2) Vijay (3) Vioin (4) Vintoo (5) None of these 50. Vilay is the neighbour of. and, (2) Vipin, Vikas (2) Vinod, Vikram (3) Vikas, Vikram (4) Vikas, Vinod (8) None of these Scanned by CamScanner yoursmahboob.wordpress.com Chapter 2: Arrangements, Vino belongs to Ghaziabad and does not play chess or hockey. The person, who plays chess, does not belong to Noida. Ram ‘lays football. Sajon belongs to Delhi and does not play chess. The person, who plays. badminton, belongs 10 Faridabad. ‘Sachin belongs to Sonipat and plays cricket. Alay does not play badminton, Who belongs to Nolda? (2) Aly (2) Ram (3) Yogesh (4) Vinos ) None of these 2. Which game Is played by Sajan? (2) Tennis (2) Badminton (3) Chess (4) Hockey (5) None of these 3. Who plays Tennis? (2) Ram’ (2) Sajan @) Vinoa () Nay (©) None of these 4. Which of the following isthe Incorrect combination? (2) Ajay ~ Noida ~ Chess (2) Vinod ~ Ghaziabad ~ Tennis, (3) Sachin - Sonipat - Cricket (4) Sajan - Delhi - Hockey (6) None of these ‘5. Who plays badminton and belongs to Faridabad? (2) Ram, (2) Sajan (3) Nay (4) Vinos (5) None of these Directions (Q. 6-10: Read the following information carefully and answer the questions given below: In an eight floor building, eight people, namely Varun, Manu, Priya, Gaurav, Tanya, Vidt, Sanjay and Reena are lving on the aitferent foors, but not necessariy in the same order. The {cpmost floor is numbered as 8, the floor below it is numbered ‘88 7 and So.on and the ground floor is numbered as 1. Two BErsOns ae ining between the floors of Manu and Priya. Who lives immediately below Manu’s floor? (0 Roane (2) Varun ) Pre (4) viait (6) Tanya 7. Who lives on the fifth floor? (0 Biya (2) Tanya {S) Reang (4) Gaurav (6) Varun (2) One (2) Two (3) Three (aren (5) None ofthese even numbered floors? (1) Tanya, Sanjay, Manu and Priya (2) Reena, Sanjay, Tanya and Vigit 39 (2) Manu Ines onthe oor (3) Tanya ves on the 4 Noor. (ss (2) 27 ais (a) 55 (6)26 after her in anh? a7 aa a9 @ 5 430 Immediate preceding” namie ‘his by tne nmber tha foe? 41362619893 8008 (2) One a (3) Tee (5) rm (2) Two (4) Four (3) Arya (6) Palak i‘ is th interchanging their clghteenth from the the row? (1) 22 ‘Scanned by CamScanner (3) Vist, Priya. Reena and Gaurav (@) Pro, Manu, Gaurow and Sanioy (5) Moni, Reena, Tanyaand Gaura¥ 10, Wich ofthe foiowing statements Is corect? "Reasoning Abiliy Exam Goalpost for Banking Exams oor of Vit and (2) Ony two person ives Between the for of Vs (4) Pry is te mediate negrbour of Vd. (5) Varun’ ors immediately above the Gaurav’ foot. 4, Ina row of gts, Valday! Is 9° from the beginning and 19*trom the end. In anothe row of gis, Spruha ls 15% from the beginning and 13% from the end. How many {ta are there in both the rows together? 32, In clase of 45 students, where boys ae twice than the number of gis, Kavita ranked 13% trom the tp. If {here are 7 eile ahead of Kavita, how many gts are 1-22): Read the following, “ sng 8-2 somes ono a Sa Eight fends rigcing away from the Centre. N is thy Bee a sae Oh Poa second tothe left of M2 28, wines: a Br ae Brno J the Immediate 19, mosatom eat ae ar £5 hone ornese oa Ic the postion of with respect to 20. Wine immediate righ (2) Tit the igh eestor (a) Secondo te {5 Tnvaro eet 24. Four of the folloning five are alike In 2 conta Based on tel pentons Inthe given arrange soferm'a group. Won Is the one that does nt terme group? (a) ne an QR (4) MR 15) 69 22, Which of the folowing pire has the fist person to the Immediate right ofthe second person? (yar (2) PR (3) oF @) RM (5) None of these Directons (Q. 23-27 Read the nfrmation and an following questior ‘tion and am, Motan Sues Mahesh, Yash and il ash and Vat are They are sitting in a straig noi same order fant line facing north, but not in Ram sis fourth to thei Of Yash. Vit does not lin Mohan isnot anim Vd Mahesh sta eda gy St eter oft seen mediate neighbour of Ram, a PS Cio ain, (3) Suresh (2) Mohan (5) Yash (4) vigie Fam with respect to Vasnp 12) Secondto teen (4) Immediate ee siting 2) ee" Ram and (4) Four a an. Dire follo Ina first is ne after there biolo 0 le ‘oft ¢ 33, 34, 27, Four of the following fvo are alike in a certain way ‘based onthe seating arrangement and ao form a group. Wh one does not belong to that (1) Ram. Yosh (2) Yash, vit (3) Monan, Manesh (@) Suresh, Mohan (5) Yasn, Maresh Directions (Q. 28-32}: Read the Information and answer the fotiowing questions: Seven trends A 8, G DX. ¥ and 2 are standing in @staignt line facing north, but not necessariy inthe same sides Y stands second from the lft end of the tie. Only two persons stand between ¥ and X. Only one person stands between A and ©. 2 stands third to the left of C Nether C nor Y ie we Immediate neightour of D. (BPS Clerk Prelims, 2035) "ehe 28, Which ofthe following statement ate regarding 8 "gy (2) stands seconate the et ore," eens {2 Dis one ofthe meta ele ours of Senay, _ (3 On one person stance beta Bn z {3} Bstnas ot one of te exveme ents eee, ime 5 Bsa ancet eee 29, Mhich of the following represents the postion of € trom thee ond of tring? (a) Second (2) Fist (3) Four (3 em San en, “*) 30. What isthe position ofZ with respect to ¥2 (2) Secondo te et (2) nme et (2) Trdtotnenge (a) Secondvetcioh (5) Inmecite rahe 31. Which of the following pair stands at the extreme Swng "ends ofthe line? Bao aa fg = OX wea tm «20 32. Whats the postion of X with respect to? sey) Immediate ign ay menace et St) Suondtomnerem — nomen ‘J (5) Second to the left 24 Directions (Q. 33-37): Read the information and answer the folowing questions: 4 In-a week, sit lectures are scheduled in such a way that the first tecture is on Monday and the last is on Sunday. IT lecture 's not on Tuesday or Saturday. History lecture is immediately after the lecture of physics, Cives lecture is not on Friday, and there is a one-day gap between the lectures of civics and biology. One day prior to the schedule of math lecture, there is ‘no lecture (a5 that day is the ‘off day, and Monday is not the ‘off day), (IBPS Clerk Prelims, 2015) 33. Which of the following Is the off day? (1) Tuesday (2) Wednesday (3) Friday (4) Monday (5) None of these ‘44. Which lecture is scheduled on Friday? (2) Math (2) History (3) Biology (4) Can't be determined (5) None of these “~_ yoursmahboob. wordpress.com Chapter 2: Arrangements 38. Wow many lectures are scheduled between math and Iistory? (One (2) Tree (3) Two (a) Cant be determines (5) None of hese 36. Which of the following I the last lecture suneduled? (2) Ces (2) Bislogy (3) History am (6) None of these 37, Which lecture Is scheduled on Tuesday? (a) History (2) Matn (2) Biology @r (5) None ofthese Directions (Q. 38-42): Study the following Information and ‘answer the following questions: Seven friends of @ college, namely Dilip, Entony. Falgun, Gheanjal, Harry, ishita and Jonali are big fans of AP) Abdul Kalam. They went to a market and bought seven novels written by APJ Abdul Kalam, namely Wings of Fire, Inspiring Thoughts, Mission India, Spit of India, The Luminous Sparks, India My Dream and Guiding Souls. They started reading the novels on different dates of the same month, viz. 4%, 58, 7, 14%, 13%, 415? and 48" but not necessaniy in the same order Isha started eaing he novel on 1° of he month but she rter read Spt of nde ot The Luminous Spc The end oread te wing of Pre stared eating on Sof re moth Fagun read Mision ia and he state reading nother on nor on 15h of he mort, Harty stared feeding 13% ofthe month and nea not ead St of nda, Te lord who food Gutng Sus tarts reading on Po tre meth lip Sane teaing neither on 38 or on 29 ofthe month an Ne did ot read Gung Sous. he rena who started reading on Sof he tanh ci ea ina My ream ona int fea Wings of Fre and stated reading on 15 ofthe month. Emory sot read Wings fF. The end who started reading on 9 ot he mont ead nee Te Luminous Spar nor Spr ot nda {SBI PO Mains, 2015) 38. Which ofthe following novels did Dilip read? (2) Wings of Fire (2) Spirit of india (3) India My Dream (4) Inspiring Thoughts (6) Mission india 39. Four of the following five options are alike in a certain Way based on the above information and form a group. ‘Which ofthe following is not a part of the group? (2) Gianjai-ts (2) Harry-13% (3) Entony.15% (4) Ishitaaam (6) Falgun-70 40. Which of the following combinations is correct? (2) Gitanjali Wings of Fire (2) Ishita-inspiring Thoughts (3) Entony-Guiding Souls (4) Jonali-The Luminous Spirit (8) HarySpirt of india 41. On which of the following dates did Falgun start ‘reading the novel? (4) 13% (2) 18th (3) 1st (4) 15th (5) 7 a ee Scanned by CamScanner folowing questions: 's sting exacy between Jand K Immediate neigntour of) oN ioe (3 Ke oure an, (8) threo four prin (5) None of these. . Ane LE, Fand G are seven members of a sport club, Al pe om na aie sitting around a circular table facing towards the ar Secalnncceaecinsmmecangens A poitminahcammmateges Blea 47, of - Besotinecwetemetenay Se gay (3) EandF (4) DaknaF aN aa (5) Ganac (5) None of these (4) Aorg 4 mmc eto ae * Mt teen ng etetamine assem a2 ‘Scanned by CamScanner cam Gospost or irae 0 . reasoning Abt (3) edite neighbours FB compnations of novel and date (8) Contymetae ie F | ne chs repect oa? (sy asim a ‘tay eect 50) Stuy the folowing (Q) The Luminous Soars - 18% Directions (0. 50 e estions: caarense T= rower th ees PuDIShINg co { Insping outs - There are Seven rw Deeika, Ela, Fagun, Gaunt. (8) hone te above aa Soe ore earn Sh 2 dean ad anew to yarn ten te’ “ ucts 9-0 te ration Seton a arian Shay ae ose a EICCMIN GP ard we sng aun ocr facing ve cere. Each of thom wos bom in 2 dierent 4197, 1975, 1978, 1980, 1981, 1984, 1985 and 1980—but nt necessariyin he same order. ts sting secon the right of Lis siting hr othe right ‘1. Land.) were brn before 2980, On the one born in 1988 siting second tothe let of. Pie not an immediate neighbour of Nis younger tran LK was bom before 0 443. Whois the second okies inthe group? ws je aK iP (2) HR Fagun -& (©) None of nese (2) HR Hema -¢ 5 nt Solution ~ Babita ~ ‘44. Who stig third tothe vgn of 07 Goer (2) Me one tem in 4870. (2) To one bom in 1980 (4) Adeicirenve- Ema 0 (@ ecne bem n 1965 (4) Te one bmn noe 1 A esc (8) None of nese 51. Which ofthe following groups of employees work 45. to wich year was R born? Content Solution department? (a)'t979 2 1975 (2) Deepina, Babita and Ea, (3) 1980 (@) 1985, (2) Deepika, Babta and Chaya (5) None of these (3) Deepa, Badia and Gaurt 46. Which of (4) Data inadequate Neen, sitements i tue regarding K? (5) None of these (2 kissing four to me rantor (2) ne ane who was bon in 1975 eon the immediate ob. wordpress.com than tree ny Ge A Ban C wth teat two in sich 98 A titoret odes wate sat the wots a Arita works 0 fo Grade A Two em es in HT department arn sre & seat son cezarment ard bones to eon nemnistratve department and does not joes content Soin in Pagan wore wit) ony Heme bel aye wars wth Ea none department, Seat wth er Chaya oF ABD inthe Sedat aun does rt Belong to grade C. Ela bobo wade A ; 50. Which ofthe folowing combinations I correc? who isthe eldest, isnot an Ris olcer than ony M. Rs (810, 2014) 52, Hama bong to wen grade? an ae Be (8 Daa nscequte {) Nene of ne 53. Gaur belongs to which grade? ing between N and K. 87. Dire ina it post 59, Dire: care Seve subj. Matt of th from ch Wea: lectu Tues on 7 on $1 subje chen yoursmahboob. wordpress. COMA: sansnens 56, ma row of students, kran’s poaltion from the laftslde (of te row ln 10h and that of Simran from right side of the row fe 23%. f Moana ls siting justin the middle of mati, Kron and Simran and his postion from Kiran le th, a ee et Mang he, 345 (a) a2 oe. oo (5) 49 =f |57, Viena ls 39% from the right end In a row of students and xin gs Geota ts 42° from the left end. Find out the total mah Sinden stgen tt my (a) 80 O98 Lopes comet atormned sett {5} Other than the given options SOT vetene (2 58-60% Sty the folowing Imation z= carefully and answer the given questions. gt % Brow of students, Vioy's 11% fom hele end, and Hemant Say is 17" from the right end. if both of them interchange their Se positons, Vijay becomes 15% from the let end. ‘58, Find out the new position of Hemant from the right end ” of the row. (a) 20 (2)19 321 (4) 22 (5) 18 59. How many student{s) is/are sitting between Hemant and Vijay? thing (1) One (2) Two (3) Three (4) Four (5) None of these 60. What Is the total number of students In the row? (a) 32 (231 (3) 34 (4) 28 (5) 26 Directions (Q. 61-65): Study the following Information carefully and answer the given question. Seven friends AB, C, D, €, F and G are studying different sutjects, such as Hindi, English, Physics, Biology, Science, Maths and Chemistry, but not necessariy in the same order. All of them attend different lectures on various days of a week rom Monday to Sunday. Out of seven, there are three female members C. the sister of E. studies Biology and attends a lecture on Wednesday. £, the wife of F, studies Physics and joins the lecture on Friday. One, who studies English, joins a lecture on Tuesday and aiso a female member. B and F join the lectures (on Tuesday and Saturday. respectively. One, who joins lecture ‘on Sunday, studies Science. A joins lecture on Monday but the subject 1S nether Hindi nor Physics. D joins the lecture of chemistry on Thursday. (2) 8. candF () B.cane {5} Other than the given options 163. Ajoine the lecture of _on __. (2) Find, Saturday mathe, Tes 3 cay {@) Hina, Fedor (3) English, Tuesday (5) Matrs, Monday 164, Who Joins the lecture of Hind? wr 2G ae (8) Either (2) oF (2) (5) Other than the glen options, 65. Lecture of __ was arranged on Monday and loctu ‘of __ was arranged on Saturday. (a) Find, Maths: (2) Maths, Science (3) Hing, English (8) Biology. Chemistry (6) Other than the given option Directions (Q. 66-70; Read the following Information carefully and answer the questions given below. 28.6, 0. EF. and G are seven members of a club. Each of them likes one day of the week, Monday to Sunday, but not necessary in the same order. Each one of them owns 2 Giferent car, namely Estilo, Swit, Figo, Alto, Optra, SK4, and Beat, but not necessarily in the same order. C tikes Wednesday and his favourite car is neither Sk4 nor Optra. E does not like Monday and his favourte car is Beat. The ‘favourite car of the one who likes Friday is Figo. The one whose avourite car is Estilo likes Tuesday. D likes Saturday and D's, favourite car is not SX4. G's favourite car is Alto, F kes ‘Thursday, 8 does not lke Estilo. 66. Who among the following likes Tuesday? (a 2B 0 (4) Data inadequate (6) None of these 67. Whose favourite car is Figo? (a 2B QF a (5) None of these 68. Who among the following likes Sunday? @a ae a0 we (5) None of these 69. Whose favourite car ls 8x4? ae 2/8 (3) wer (5) None of these 70. Which of the following combinations Is correct? (1) F, Thursday, Estilo (2) C, Wednesday, Alto WS —_ fois inre sa Teoasiy ond’__joles eine (3) D, Saturday. Beat (4) G, Monday, Swit ‘ ss bemaey, —_— (5) None of these 6.8 ‘ mee ana 1 {atin ¢24t rom the let end in a row of 36 students. ’ 5 end? 1) Nino hese iis oie 62. Who represents the group of female members? a2 20 (1) A.Bangc (2) C.EandG (5) 22 ens 43 pe Scanned by CamScanner Reszoning Ability Exam Goslpoe for Boring Exams 72, the postion of Karl fam the tet sido of # ioteabe sented a etn a 7 25 aa ae 63 Oa ate 73, In arom of students, Sait is 13th from his oft end of {he rows. When shied to his right by 3 places, he becomes 26th from the rg nd ofthe row. How many students are there lathe tow? ize 2) 28 Glas (458 os 74. tna tow of 86 students, Abhishek Is 46th fom the end. Flog out Abhishel’s rank trom the staring poet ofthe cat as (st or en 75. In the sequence 43.685 79 246,11 fist and second Alig are nterchanged, then {nterchanged ‘and ith an IMerchanged and so on, wich digit outa ‘sevonth trom the rch? ws ae 2 ae 4 Ditficutty Levek: HIGH andanone i toSE Ret the owing tomato carey End anawer the questions gen been namely 1 M,N, 0 and P veto Moe J studies MEA but Berson, who studies MEAs th at Watching the movi in sereen 3, The ‘owe in the same Seren, in aching the nen is wang Kase ge 1 wating he moves ne sod te sty vo sro goto st Sx fee ca soon Wi aye tM ‘CH hk encanta © Whit couse des cays eas (218 Ten Glen (ee men 2 Wha states one aa ae om as ae 3% Which ofthe follow roi mn ops we wing Ge wenan 2) Ma GI sinanat Ronny (5) N. Panam . ‘Scanned by CamScanner ery en at intact ete Zesiraramran mmm aoe a eet toning soc eolntn ws ayes: seoen2 , (3) 6B. Soren Brea: sacen 1d Ie watching 9 MTech 0 does study © Seen ar an (ao an 4 weg. e105 Read the follwing Ina Directions (2. below: oe questions gle cate an omer te es and G purchase, Seronponone nae 28 60.6.5 002 DUCE oS ett es ly necexsariy i ne sme ode. Each, pve es 1B the cae win cere cris, but ok mecesserty green, golden, black ‘Same order, Out of seven, fur persons 1 car but does not pur ines the green col nt Rare tno person who purchases the \20 ear the nhte olor and ise female. E, te Sstr of F, pure the guden cour Ato car @ ikes the Brown colour anew ‘BRIO car. F, the wife of C, purchase the Car ond hes te siver colour and C Ike the white an ‘ete car rexpetey The person who purchases the Seq tee the viet our 6. Who purchased the car Swift? aa 8 Be we ot 7 Which ofthe following isthe group of mate persons? (D5.G.0F 24,006 CORE @aBGo (5) None of these 8. Which cars purchased by a? 2) Scorpio (2) swine @) Ato ) sxa (5) Eon 9. Who tikes the colour violet? o 26 a ar ae 20. Which of the fotiowing {2 Male on een #8 ncorret combination? ) Femaleit0 Si the tm and 27th rom an examination competion ang gaa re inthe aces 35. 36. 20. =~, fit a meet ine oe Shia ahF; efi 31 a) 32 {51 None of nese 413. Ina class of 45 students, Pravin ls placed 8* from the ‘bottom whereas Savita Is placed &® from the top. Kian {a exactly In between the two. What le Kira’s postion {rom the top? (2m (2) 18» (3) 23° (@) 13" (5) None of nese 414, In a class of 90 students, where girls are twice the ‘number of boys, Samadhan ranked 44” from the top. if there are 7 boys ahead of Samadhan, how many girs are ater him in rank? (se (2) 28 (3) 22 (a) 36 6) 40 45, Ina class of St students, Viay ranked 20 from the top ‘and 11° from the bottom In a competition. 13 students {id not participate in the competition. Find out the ‘numberof students who faled the competition. ait (219 a? (4) 10 os 16. In a class 0 ranks 8 from the front and P ranks 10 from the last. If Q Is at 15% rank after O and ts exactly Inthe middle of O and P. Then how many students are there in the class? (a) 43 343 (5) 48 17. Find the number of boys standing in a row facing towards north, If Himesh ls 12% to the lett of Prithvi, who Is 279 from the right end, and Mayur Is 23% from the loft end and 98 to the right of Himesh. (2) 46 (a) a7 (a) 47 (2) 50 (3) 52 (4) 53 (5) 43 48. If the first and tenth digits in the sequence 6 8 9753 5 4.9 1 are interchanged, followed by the second and ‘ninth digits, the third and eighth digits and so on, which dight would be on the fourth right of the third digit from the left? ws 26 7 a8 69 19, If sanlya finds that she is 16% from the front and 8% from the back of the queue, then how many giris should bbe added to the queue such that there are 35 girls in the queue? (2) 10 (2) 14 (3) 13 (a) 12 (6) 15 20. Six dress codes, namely P, Q, R, $, T and U are to be planned in a span of seven days, that is, from Sunday to ‘Saturday, only one dress code each day. P should not ‘be planned on Thursday and R should be planned Immediately after U. There should be a gap of two days between T and S. Q should be planned on Tuesday and should not be followed by S. If one day there will be no ‘dress code (Friday is not that day) and Just before that — _—————_ Scanned by CamScanner Directions (Q. 22-: Carefully and answer the questions glven below: ty Sik frends Atl, Mot, Deeparshu, Bhovesh, Soham anc (i) Allthe sides ofthe table have the same len. (ii) Atutis not edjacent to Deepanshu or Mud. iu) Arnay sin the cdl of Bhavesh and Deeparshu. (0) Bhavesh is not adjacent to Deepenshu or Soham. (vi) Mudit and Deepanshu are adjacent. 21. Which of the following pair of friends are not sitting 22, Which of the following 123. The distance between Bhavesh and Deepanshu is same 24. Who among the following are the Chapter 2: Arrangements day S itt be planned, then how many dross codes are planned between U and 5? (1) One (2) None (3) Two (4) Tree {5} More than three 5): Read the following formation Jrnav ore sitting around @ hexagonal shaped table facing towards the centre 2 ana Sonam Seurona Soham @) Muctand . (2) Auoragnavesh (a) Deeporshu and Soha {) Arnav and Deepanshu fe sitting In the right sequence? (2) Atl, Armay and Must (2) Armav, Atul and'Soham (3) Muat, Deepanshu and Armav (@) Bhavesh, Atul and Mudt (6) Armav, Bhavesh and Mucit as the distance between Atul and (4) Soham (2) Mudit (3) Deepanshu (4) Bhavesh (6) None of these Immediate neighbours of Atul? (2) Mudit and Soham (2) Deepanshu and Mudit (3) Amav and Deepanshu (4) Bhavesh and Amav (5) Soham and Bhavesh 25. Who is sitting exactly opposite to Bhavesh? (4) Muait (2) Sonam (3) Atul () Armav (6) Deepanshu Direction (Q. 26-30. Read the following information carefully and answer the questions given below: Six friends Garima, Deep, Shaan, Mehul, Sujata and Aliya are sitting around @ circular table facing towards the centre, but Not necessarily in the same order. They have different professions doctor, engineer, lawyer, scientist, teacher and Singer. They are wearing shirts of different colours white, black, blue, yellow, orange and green, (i) The persons who are doctor, lawyer and teacher wore neither black nor white shirt. (ii) The persons who wore blue and orange shirt are neither doctor nor lawyer. (ii) Garima is neither in white shirt nor on the immediate left of the scientist. 45 for Banking Exams ‘Reasoning Ability Exam Goalpos| ‘nota teacher, engineer Shit andto the immediate ef of the lawyer (Ai0 Shaan isnot. a lawyer and Aya ie nota doctor. 26. Who among the following wore a white shirt? (2) Garima’ (2) Shaan (3) Menu (4) aya (©) Dewp’ 27, The person, siting between Sujat and Mebul, is oaringa shir of whlch colour? (2) yetow (2) green (3) beck (4) orange (6) blue 28. Who among the folowing isa teacher? (2) Garima (2) Shaan (3) Menut (4) Alva (5) Deep 2 MMCH ofthe flowing le conecty matched? (2) Garima - orange “selerus (2) Deep ~ Yeliow = Laven (3) Shaan - white Singsr (4) Suista- Yelow — Doster (5) Aya stock Teaches $0. The colour ofthe singer's shin is (2) yetlow (2) orange (3) green (4) wnte (5) black fotowien® (9 34-35% Read the ivomation ang ‘answer the following qu levees ofan office. They ate siting together around a eveuse contre) inthe office, @ (299 aw ar oP 32. How many persons are sitting between R ‘and V starting (12m R in an anttclockwise drectiony (2) One (2) Two (3) Three 4) Four (8) More then four 33. Who ls seated third tothe ltt of Pp (ar 219 - @R ‘Scanned by CamScanner Oob.wordpress.com me Jota and Ada ‘engineer is sitting bet Pu ‘in white shirt pmontotennettsceatbe seen (5 facing the (‘Mehul isa scientist and isn be shit. He facing nin green shit, (1) The doctor i siting opposite to the person i ena hile the person in be shits tothe left of the (4) The singer's tote immediate right ofthe person In white ‘the left 7 (6) Four five are alike in » lowing 35, Fou te itlng. Doane ia bastjement and 50 fOrM & ETOUP. Which og belong to that group? 4 ve (a) RO (a) wv (3) Ps. w 2 (@. 36-40}: Read the information ang, Directions (Q. {elon ee and Zar siting around a squore same order Hote ery nthe some o the conte, not Pape table while the remaining st if at cm rates, Yistirdto the enor Wie Cen orAnno escordo the eto Big tear wnols tro the right of X right of O who (IBPS Clerk Pro mela lt of v2 28, he iting te inact 80 A oe we aang ote et tz Be ae ‘ A 38. Whois stingto the Immediate right ot A? ae oe Sx (4) Data inadequate Bien tn 29. smatieZs poston vin eget oop Breen eettoge grmasece 2 Meo mein a aneoeat ‘ela iatsae ent pnag Sizimedonat ae ft or ce (2) x; 8) py au am Directions (Q. 41-45). answer the fo Mizoram In tow 2, the Delhi, Bihar MS, inthis OW faces a Ministers of Assam, Meghalaya are seat ‘arrangement, member of tne faci 20h mem other roy, Td ing Tit Miser ot Asso sng th Minister of Dah. Nether es Chie the Chit Minister of Dein sng extreme get OF Mura 8S the Cie nae ® ree " Chier feceniNahal Pradesh doe niet Dt The chier mi ASSOM and the Chief Minister {20 ty 'aniour, h rd as or of Jharkhand sits at one ofthe extreme ends, Only two Ministers Sit between the Chief Minister of narkend and ghar. The Chief Minister of Bihar oes not face the Chief © Minter of Arunoche! Pradesh. Two Chiet Ministers sit between, ‘he Chet Mirister of Nogaland and Tripure. he Chief Minster GF Nagaland is not on immediate neighbour of the Chief Minister of Mizoram, The Chief Minister of Manipur does ne face the Chief Minister of Arunachal Pradesh, The Chief Minister of Sikkim is not an immediate neighbour of te Chief Minister of Nagaland, (SB1 PO Mains, 2015) ‘41. Who among the following sts at the extreme ends of the row? (2) Chief Minster of nartiand and Bihar {@) Chief Minister of West Bengal and Mizoram {@) Chief Minister of Skim and Negalend (@) Chief Minister of Jnarkhand and Meghalaya (6) None of these 42, Who among the following faces the Chief Minister of Assam? (2) Chiet Minister of Nagatana (2) Chiet Minister of Mizoram () Chie Minister of Sikkim (4) Chiet Minister of Tripura (6) Chie Minister of West Bengal 43. How many Chief Ministers sit between the Chief ‘Minister of Mizoram and West Bengal? (2) One (2) Two @) Tree (4) Four (6) None 44, Chief Minister of Meghalaya faces Chief Minister of ‘Atunachal Pradesh. If the Chief Minister of Sikkim is related to the Chief Minister of Arunachal Pradesh and the Chief Minister of Manipur Is related to the Chief Minister of Meghalaya then following the same pattern which of the following Chief Ministers Is related to the Chief Minister of Bihar? (2) Chief Minister of Jharkhand (2) Chief Minster of Dethi (3) Chief Minister of Manipur (3) Chief Minister of Assam 6) None 45. Which of the following. is true. regarding. the Chief Ministor of Meghalaya? (2) ct nse of Megas econ the tet the Chit Minister of Manipur (2) Chief Minister of Meghalaya is not on immediote feignbour of the Chief Minister of Assam (3 Ger of Megnaay ttt the ‘Chief Minister of Det “ (4) Ghiet Minister of Meghalaya sits at one ofthe extrem fends of the line left of the (6) hone ot nese rections (0. 46-50: Study the Iformation and answer the faleing questions: Spans 0, EF 6 and Hae standing in 9 sae tne face morn bu ne newssary in the same oe D is roe resondto ne rene cis standing fourth othe et cintand ts not sanding on te exreme end of te Ine, Eis Standing second tothe right of D. (SBI PO, 2014) 46. What s the position of G with respect to? (4) Immediate let (2) Second tothe left {@) Third tothe left (4) Third to the right (6) None of these 47. Which of the following pairs represents the people standing at the extreme ends ofthe line? WeH Qe @ OE @cH (6) None of these 48. Who Is standing second to the right of C? we 2D @s @e (6) None of hese Four of the following five are alike In a certain way based on thelr positions in the above arrangement and 50 form a group. Which ofthe following does not belong to that group? ace QGe @GH @) DE FD 50. If all the persons are asked to stand in an alphabetical ‘order from left to right, the positions of how many will main unchanged? (2) One (2) Two Q) Three (4) None ©) None of these 47 eee ‘Scanned by CamScanner a Bfe]=|sJolalaluls lle mi Tres ltt. ‘Scanned by CamScanner ond 38. socording tone, 18696-58 iow, opoly the operation on 54 ond. ‘According tore (), 54x 3=162 ay So the resultant value of row is 362 Fe 4, On vm poms a Fi 90 the operation on a ‘According to rule (), 4 99 +333 XN Now, apply the operation on 33 2 and 38. ) a B15 2] ue of row So, the resuitant value of row Sy) ismasz Bal rorrow2, a] estesin the vous of Zin ow 2, a the row becomes, ay 603 @ wl ‘Apply the operation on 15 and 6. ol scording to rule (iii a] According to rule (i, P 15-6=9 a ‘Now, aplythe operation on 9 and EE 3 ay According to rule (0), a a 943-3 oa So, the resultant value of row 2 is 3. 3. (8) Forrow 1, First apply the operation on 369 and 41. According to rule (i), 369+41=9 ‘Now, apply the operation on 9 and 2. ‘Scanned by CamScanner According to rule A, 9-297 So, the resultant value of row 2.8 oi For row 2, First, appty the operation on 245, ‘and. According to rule 285 + 5=49 ‘Now, apply the operation on 49 and’. According to rule (), 494727 So, the resuitant value of row 218 7 The product of the resultant value of ow 4 and row 21s, 7x7 =49 (5) For row 4, First apply the operation on 2 and 16. According to rule (i), 18 Now, apply the operation on 18, and, According to rule (iv), 24 18x5=90 So, the resultant value of row 1 is 20. For row 2, First, apply the operation on 22 and 3, According to rue (Iv), 22x 3-66 Now, apply the operation on 66 and 36, According to rule (i, 66 + 36= 102 So, the resultant value of row 2 is 102. The sum of the resultant value of row 1 and row 2is, 90 + 102=192 ‘According to rue (i, 42+16=58 ‘Then, apply the operation on 58 and? According to rule (), 58x 7-405 ‘Then, apply the operation on 406 ‘and 484. According to rule (i, 406 + 484 = 890 -Tnen, apply the operation on 890 and 3. ovording to ule (). 890 x3=2670 ‘so the resultant value of the row is. 2670. 16. (4) Vishal ranked 5” from the top ‘and 46% from the bottom of the class. Therefore, Total number of students = Number of students ahead of Vishal + Vishal + Number of students behind Vishal = 4 +1. + 45 = 50 7. (3) Total number of girs in class = (Number of girls ahead of 27% girl) + 27 git + Number of gis behind 27m git = 26 + 1+ 26 = 53 8. (4) Total number of students in the class = 41. ‘Suvarna's rank from the bottom = 41 -14+1=28" ‘Saugat’s rank from the bottom = 41-174 1225" 9. (1) Total number of students in the class = 49 Disha's rank from last = 25% ‘As Navin is 11 ranks ahead of Disha therefore his rank from the start = 49 - (25 +11) + 1= 14" Fe sry xam Gosipos for B 42, (1) Let's rovers te nu ‘ert Tere mobo Bee ate 2820 that Is, 16th number, da Pesce el me Serer (a) 13. (4) From the representative figure "Nissiting between M and 14. (1) From he representative figure, ‘afer interchanging the position of Q and RR wil become. the Immodiste neighbour ofS and. 25. (2) From the representative figure, after interenanging the postions Of Q and P. Q wil become the immediate neighbour of Rand N, 416. (3) From the representative figure, SR and P are siting in a sequence, 417. (4) From the epvesentatve figure, Sissitng between Q and R ForQ. 18-22: ‘According to the given information, the Positions of six bys are shown below: ‘Scanned by CamScanner be rem (a7 iP inden ome anking Exams Jana fom et ee veo " prmrmenene germs ee rere ” Rajendra is third from the lef speme - gerne arcuarrere's i cede te vatve gue, erent Seems place in the cass from thetop=9+7=16 Seema’ place in the cess from the bottom = Total students — ‘Seems pace fm the top + 4 = 35-15+1=20" 4 (Q) When the postions of two Persons are given andthe, Postlons are interchanged, after interchanging postion of ist Berson is. given from the same Sie as befor interchanging, than Postion of 2nd person from the Same side = Postion of 2nd Berson from same side before interchanging + (Poston of 1st Derson ater interchanging = Position of st person before interchanging trom same side) Therefore, Postion of Styam from the right fend ater interchanging = Positon of Shyam from the right end before interchanging + Position of Mohan after interchanging) = Position of Mohan before Iterchanging poston of Shyam tom Era ater nerchongng= pe peat Therefore, the new Sryom is 22 rom ene oral no. ot st - etion of Soten tom (Satin of Some rom ia Tota. of ders Nps a9)" rota. of tents = 43 (2 Ken's rank trom tops Ge Dac; teretore, is ra Bosom is 42-232 = 200 a 1 (8) Number of persons bet ‘Arun and Jay = 56 ~ (16 + 4¢ 21 Now, Mahesh les in middie these 21 persons, it means he ‘t the 11th place so, the num Of ‘persons ‘between Arun a Mahesh = 10 (4) By using the formula, Total = Top + Bottom - 4 Total = 15 + 29-4 Total = 43, For Q.29-33, SNE te shen ifrmation Beno ang“ O® tale pia cast tniacinve ated ne Pah toate vlvaweaci FRE Goloediin rena : For Q.34-38: (4 According tothe representative figure, Ankit is Immediate rignt of anit. 43, (3) According to the representative figure, wih the help of Interchange Indication, Ankita fs ‘second tothe left of Aman. rera.aeas of =. aa ioe fo te | we so, [ame Jose Tv stom [ef omer oe ‘Black T Anos so woite ae. rey = =. = 44. (5) Number of students between Mone ae Ygetn 290722 34 Aecrone ote orange, Ma feva ties he yon = 4. (4) In such type of cases, when 235 2) scorarg toe aangonent “fa sits" se 8s Ben reno sors es tom te same seston (a too ra we ent td ok 38 2 sean othe ange, a, ieee be am total nomirofaete ‘37. (4) According to the arrangement. For Q. 46-50: eal Ankita is a CO, who likes. the we 2 coowr rot 38. (4) According to the arrangement, “Vipte : soutien coup ~~ oe oe rg 9-48: - Se ics ne 48. 4) In seh pe of austin wren ong of penne ex cs numbers tend eer ne Sa Cann ed amon’ not ot £ boson. ts question, there 39. (2) According to the representative figure, Ankita sits immediate rignt of Amita. 40. (5) According to the representative figure, Amit sits between Ameen and Amina, 41 (1) According to the representative figure, no one sits between Annu —__ ‘are 7 persons, which is an odd umber. Thus, we cannot ‘determine the in front-of position, 47. (2) According to the representative figure, Vikram sits between Vinesh {and Vinod, (6) According to the representative figure, four persons are sitting between Vinesh and Vinod, if we Start from Vinesh in the clockwise direction, 0, (6) cording ‘eve, ay m nogrtow ot MB Vio, Ditnculty Lovet: MODERATE Fora. a-et " fecordig tothe gen infomation, the ‘Govoncipemeng te person, and tome ar town tow be Ram_—_| Noida Foot Vinod | Grasiaved | Tennis Nay __| Honour cess secnin_| Sonat riche voresh | Fandabad | Badminton Sain | Dem Hockey 4. (2) From the representative table, Ram belongs to Noida. 2. (4) From the ropresentative table, ‘Sojan plays hockey. 3. (3) From the representative table, Vinod plays tennis. 44, (2) From the representative table, "Ajay - Noida - Chess ‘combination is incorrect. 5. (5) From the representative table, Yogesh plays badminton and belongs to Faridabad. For Q. 6-10: According to the given information, the relationship between floors and persons are shown in below table: Manu viait Reena Priya Tanya Varun Gaurav ‘Sanjay 6. (A) From the representative table, Vidit lives immediately below of Manu's floor. ‘Scanned by CamScanner 54 7. (Frm te representative table Pr ives on the th foo. 1 rom 9 meet ttn, fo ponors nae Yo and Sour ne ttc he oar te Rx on whch ey (2) Fo te rere able Mou eer Tana on Garey theorocr nentoed Tos 410. (3) From the representative table, Tanya ives on te 4 foc, 14 (3) Number of is in Vidas ow wards t8e2r ‘Number of gts in Sputa's row = uasteazear *% (2etus assure ¢ dros acarand 8 dence abe Visuals 2 paring lt as pr the ghen sequence. Reasoning Ability Exam Goslpost for Banking Exams ‘otal number of gis in ott rows = 27 +27 = 54 42, (1) Let the number of girls be ‘and the number of boys be 2x Total number of students in the dass = 45 x of its in Therefore, number the class = 15, and number of Doys = 30 ‘Kavita’s rank is 13" and 7 girls are ahead of her Number of gis behind Kavita=15-7-4=7 (2) The betow figure m3 There ate two oven which are divisible immediately numbers but not divisible by the num respectively follow them, 81362 6ip 8035e, (8) Tat rurer of owsone gueve= 60 ne Number of people between tant "6019 3030 As Pooja Is exactly between two therefore her’ postion 1a. ea tae p29 22113 241[26/17ra2{20f21]22123]242] SEES EEE Bag@qg Onna Sof rete ee foe to te SPS Sefton soa feos at petesleet eh BEbGbb pick lel h heroes eee az Fm atte. can eet tee 38 es the second ar of orc * eiamne 1 te. gwen 48.) tn oe ‘eprecenatie 25, (2) From the arengement above] 3 information, the sectors of man arrangement tgure, Pi siting Suresh, Yosh and. Mohan {sae shown boo secondo the leaf Siting Between Ram and Vit. 3 septa Oe net yi setae 28. 4) From the arrangement ato ta as arangement figure, T sts o the Ram and Mahesh are siting at tne ane immediate en oN ‘extreme ends ofthe line. wv rae % (from the repesentthe 27. (5) fom ne arrangement a a wo ea rangement fee. Tt ted to fxcept Yash and Mahesh, there eletore Pi aac one person seated between Se m2) Fem tm itpiiodeang ‘0 of al other pairs. 3 arrangement figure, exces Ma a * iccremetsevemin eMail puget font and Shavesh Rone between thm, a Boston is 189 rom ie 2nd postions of seven na fore mat 22) Fam the "epcesertatie Shown below, ne arrangement fgure, Ns arn 13a! pmber ote = ‘heimmedieterght oft, NE 2 For Q. 23-27, : FG. dp-2n Accord WE to the given information, tne she siting rangement of eight ends Seating arangementis as folowes ” © | oven beac MoKdedinometon P is even below pe {0 th ne e 3°trom 4 rent monn eRIntbe ‘Stan ty ye Of the In, ; Seating arrangement one person gt. ss I 4 between B and rangement above, ag 12s aston of Ram oth res &) Fro = ‘0Yashs second tothe len 38, 52 iz = ‘Scanned by CamScanner Bie ak.) fom On atm anargene A.) chan got Fe a Seb ie renee Cantante mona ines a crac, $2. (1) From the above arrangement, ne is on the immediate right of C. 42. (5) Hary read The Luminous mht * ‘ esee Sparks on 13% of the month Contentsouion ©) 694 Fo For Q. 43-46: os ‘herrea pee ep arin Se cast! 4 ot = te "sing Sacre” meeeermemmennaes [nese |eameonen [> | Pear ys shown blow {ound a ores tae oon = aaa 1 og = =e 1 ae ante Tuesday e | Gouri ‘Content Sotston 350" weanessey | Civics Hema | HR _ = le So (Te contnaton Mie based Soe ive ~ Ella ~ Cis correct. sty [eso for : 4) botin Down and Gait Bal Sunday mavenoics | = works in Content Solution te} 43. (2) Lis the second oldest in the ‘camels 38.9) 6 por te aioe uve F arn Sut ecco SoiparosssbanieSSTa, 5p. cynane alongs we, Gi tarwtectowan aay HE EDISSRUEIRTOME HERO. (home mgr are lecture is scheduled on Friday. ae 54. (3) Chaaya belongs to grade C. ‘38. (2) As per the above table, three 45 (4) R was bornin 1985. 55. (3) Varsha is 26th from the top om leurs (cna, Tandbone) we 48. Ke tinea oe ator SE vane 26 fom te op me sealed between mamenous —“* Sthecne wast nore ada 26+ 9 = 250 fom trun eee Weonteimmmdserenstk ——-Waguhanmsngon te bam etic tet «Romp Kony wes «(Mae ean, we 88 person. vong ene ones evel aa Be mathematics. ‘between Nand K. ‘examination = 35 + 32-1= 67 bad 37. (1) AS per the above table, history See ~ae (ereissmedsctorteesst” — fesorng to te fen ifraion he Therm, ttl umber of scsi orogenent ate eral Ga For 38-42 tte te pr slows ~ = 2 = (66/2)x3 = 33x3=99. '* | mse co + Cee Sf tour pi or ls ° Posten of Kron fom lf & = lewis Poston of Meera om Rat 4 ous seria, ee Se Now, postion of Meena fom right Falgua Mission India 1g” € Ie = Position of Simran from right + ston of Meena rm rane iinet [wage | ae Rostien of Meena om Kian = 23 ‘ etuninae | pr eee [= Xs Tt camber tee = So 47. (1) Tenegnbous ofAare € and __pstlonof Meena om both eds eam Total numberof tudes = (+ aidegios 48. (3) Fis tothe second let of D. ns = ( 19)-1=50-1=49 49. (5) A is to the immediate right 38. (4) Dip read inspiring Thoughts. of. 87. (4) In such type of cases, when ranks of two persons are given at 39. (6) Except Falgun-7® all the other ifferent positions from both sides ‘Option is correct. Falgun started that is, left and rig cannot Feading 18 of the month. at A Sra ee _ 53 — _ Scanned by CamScanner ing Exams : |Goslpost for Banking ~ Reasoning Abilty Exam to the arrangement j of 0A. Ch) Acco oe and on E fd out the ta number Pore be | mae San ee othe angen ao 65, (5) According to the eorene ‘arranged on ure of Mas i aranged 08 Monday ed lecture of Hind arranged on Saturday. For Q. 66-70: on According to the given informati Iiaoreip tetween mentor, dey and cars as follows: onchange of te penton yt fom teatna putea 7 amr Ths, econ att tere 8 steno tet on ere Vioyané Wyse 6 srs rome on Tota numberof students in row = trieige3t 58. (3) Now, Hemant is at ise Postion from the left afer an interchange of postions in a row Of 3 students. Thus, we can say there are 20 stunts fom the ete end before Hemant Rank o Hemant from righ = 31, ~ 10-21 5. (3) According the questo, Number of su 66. (2) A thes Tuesday, Hemant and Vioy. 3, 67. (2) Bs favourte car's Figo. 8. (4) Ekes Sunday, 69. (4) Fs tevaurte cars xa, 70. (5) Al the provi {te incorrect. 72. 3) Lert dents between 715-114. 80. (2) Tota! number of Students in the rom = 141 36 31 ied combinations 4, Total = 35, Right => Total= Left + Rignt 4 Right Total -Lent+3 "35-2461 -2 72. (3) Total number of persons = 15 +47-2- 2099) gage 78 (2) Tota nao stusets = Sum of same person trom 62. (4) According to tho ‘arrangement, feng andErencesent the gone, female members, 83. (5) Accord A joing 8644-46-87 Goagy 75. (2)The given Sequence is; Ng tothe arran igement, the lecture of 4388S To 246 mats on Sequence after interchay Gigits according to condition: ning the the given ‘Scanned by CamScanner ianbOO wordpress.com 348675296 Therefore, required sige the rent = 6 Difficulty Level: HIGH ForQ.1-5: According tothe given informa relationship among. the courses, and screens are 1. (3) From the representative ta Mstudies MBA, 2. (5) From the representative tap P studies BCA 3 (4) From the representative tabi {he group of persons P, L and N, Watching a movie in screen 2. “ 2) From the representative tablgl ia ik = BBA Screen 2° is ty Correct combination, th, yoursmahboob d PTESS.COM) amare ong ton tesa se, mamta sso ng 9 Fara Oa cet arngeent of nde : ro Te san ote maton a eer eciit n, a i cnet mee eel a Linon mepmreie ie, Rint aerP 9 coves in Dives the vols Number of students between O- sa cymommerpeanane ti, OPEL orem reese owe me tet sumer of sce oe eee ] rea enone, Laon de ones = aoe 4 sa goimumorsioon ote a, oy huni 2m rm abet Cuando ae ana _ | repamigsimsrneame Fa a mate on eine So, Himesh is 39 from the right. ‘nt i Seip be i | = Reser fee ten evn verano namely 0, P and Q so that there So, Meyur ts 30% from the right. are the minimum number of ea pnves baemeaal tsar a 29 fom om 2 persons in the queue: ‘Therefore, the minimum number ‘of persons in the queue = 4+4.+ 241+25=33 13. (3) Total number of students in the class = 45 Pravn's position from the bottom ‘= 8m and from the top = 45 - 8+ 1-38" Savita's position from the top =e Kiran’s rank from the top = 230 14, (1) Total number of boys in a class =90 Number of boys = 30 and ‘number of girts = 60 ‘Samadhan's rank from the top = 140 Number of boys ahead of ‘Samadhan = 7 Number of girs ahead of Samadhan = 14-7 -1=6 Number of girls behind Samadhan =60-6=54 45. (5) Total number of students. in the class = 51 Number of students who Participated in the competition 51-13=38 ‘Therefore, the number of boys in the row = 30+ 23 - 1= 52 48, (3) Given sequence: 6897535 491 ‘Sequence after interchanging the digits: 1945357986 Required digi = (3+ 4)th from the leftend = 7 total number of gis in 6+ 8-1=23 Number of girs need to be added to make 36 girls in the queue 35 - 23612 20. (3) According to the information provided in the question, below is the plan for dress code ep Monday No dress code Tuesday @ Wesnessay | Thursdy u Friday R Saturday P From above, we can see that two dress codes are planned between SandU. ‘2a. (4) AS per the representative Sreangement figure, Deepanshi fond Soham are not siting together. 22. (3) AS per the representative ‘arrangement figure, Mucit Deepanshu and Amav are sitting inthe right sequence. 23. (2) As per the representative ‘arrangement figure, the distance between Bhavesh and Deepanshu is same as the distance between ‘Atul and Mudit. 24. (5) AS per the representative arrangement figure, Soham and Bhavesh are the immediate neighbours of Atul 25. (1) As per the representative arrangement figure, Mudit is sitting exactly opposite to Bhavesh. For Q. 26-30: see Dea a oa Scanned by CamScanner 26. (2) As por the representative ‘arrangement figure, Shaan who IS ‘an engineer Is wearing @ white shir 27, (4) AS por the representative ‘arrangement figure, Garima Is siting between Sujata and Mehul {and she fs wearing a orange shit. (2) AS per the representative ‘arrangement figure, Garima is 2 teacher. 29. (4) AS per the ropresentative ‘arrangement figure, Sujata is 2 doctor and she is wearing a yelow shir. 30. (5) As per the representative arrangement figure, ‘Aya. is 0 Singer ands wearing a black shirt. For 9. 31-35; Based on the given information, the seating arrangement is as follows: 34. (2) As per the arrangement above, is seated between Uand S. ‘82, (4) As per the arrangement above, P.U,Qand Sare siting between R ‘and V when we count the ‘employees in an anticlockwise Airection starting from R. 33. (4) As per the arrangement above, Tis seated thir tothe left of se Scanned by CamScanner Reasoning Ability Exam Goalpost for Banking Exams toe, sper te sangemen 3 pon tt th espe ‘teu tote le or nt aove 4 As parte aang 3 rate po wera 90 Smpwyes ating. between the Smpeyee oer pre. ForQ. 36-40: ‘According to the given information, the ‘seating arrangement is shown below 36. (1) As per the arrangement, 2 is siting othe immediate left oY. 37. (4) As per the arrangement, C is siting third tothe rig of, ‘38, (2) As per the arrangement, W is siting to the immediate right of A 39. (6) AS per the arrangement, Z is Siting fourth tothe left as well as right of 40. (3) As per the arrangement, D is sitting onthe immeciate right of Y For Q, 41-45; abe themed td TLET LT 41. (3) The chief ministers of Sikkim ‘and Nagaland sit at the extreme ‘ends ofthe row. 42. (5) The Chief Minister of West ‘Bengal faces the Chief Minister of Assam, 43, (2) Two Chief Min between the Chief Mi Mizoram and West Be are the Chief Ministers of and Arunachal Pradesh, 44. (1) The Chief Minister of related to the Chief Mini Jharkhand since there ae Chief Ministers between the Ministers of Sikkim and Pradesh and also. two. Ministers between the Ministers of Manipur Meghalaya. | 45. (5) The Chief Ministers Meghalaya and Arunac Pradesh face each other. For Q. 46-50: The seating arrangement of six inaline is shown below: 48. (3) Gis third tothe left of E, 47 @) © and € are ating at oxreme ends ofthe tn, 48. (2) 's standing second ‘ight of C, : be 49. (2) In all the options, except ‘6 here is only one person stand 9th, = yoursmahboob.wordpress.com Chapter 3 Classification Scanned by CamScanner ‘CLASSIFICATION AND ODD ‘ONE OUT sriteton end od OPC Cu questions. sim to ge the Sasa’ Information processing capably, creative Dunking read evan si. ston means cotegorsing tems in cetaln groups ass conan fetuses. In the Cisaication ond Odd One based don © candidate ven certain ems wher al except a etnest the similar GrOUD. The candiéae is requded to 25 peo tem hat des note to te dassfeation ‘some aspects on which classification questions are designed In this category, al options except the odd one have Qiuar meaning. The odd word may appear completely ‘tovont when compared tothe other options, For example (2) Brave (2) Courageous (3) Smart (4) Heroic (6) Daring Inthe given example, all except ‘smart’ are synonyms. Name of subject/person/entty: In this category, all options ‘xcept the odd one relate to each other in some manner. For example: (2) Babar (2) Humayun {@) SherShan (@) Akbar (5) Jahangir In the ven example, all except ‘Sher Shah’ are Mughal emperors. Letus take another example: (4) Macbeth (2) Hamlet {@) Lord of the Rings (4) King Lear (6) Julius Caesar Inthe given example, all except ‘Lord ofthe Rings’ are works of Willam Shakespeare. ‘Lord of the Rings’ is written by J. R. Tolkien. ‘Odd.even: In this category, all the options except the odd one ‘are either even digits or odd cits. For example: (a) 34 (2) 26 @)27 4) 28 (6) 22 Inthe given example, all except ‘27" are even digs. Name of subject: In this category, all the options except the (04d one belong toa similar field of study. For example: (4) Climatology (2) Geomorphology (@) Oceanology () Glaciology 6) Biology In the given example, all except ‘Biology are branches of Geography. Class or category: In this, all options except the odd one belong to same class or category. For example: (1) Elephant (2) Seal (3) Whale (4) Man (5) Hawk ‘Scanned by CamScanner Pace: in this'll Spon éicept the odd one denote similar ‘ype of places For example: (2) Madagascar (2) Thailand (3) Cubs (8) Greontana (5) Tasmania “thailand ae stands. In the given example, all excent ne denote similar Disease: In this, all options except the odd o types of eiseases. For example: (2) Goiwe (2) Typhoid (G) Ricks () Anaemia (5) Scary In the. gen example, all except ‘typhoid’ are deficiency diseases. consist of Usually the Classification and Odd One Out sections consist 2 group of toplespacic words, gven in a set of fv Four cio re star exc erin a certain annet Set snare ine sare cnoracersica as others Thus seoades a easn for bag on ods entity. The cancaate needs fo dosly the simor ams and nate odd one out Let us understand the concert of lassen and od one oat te folowing theta: Iustration: Choose the word which is different from the rest. (a) Tonle (2) Snake {) Chameleons. (4) Crocodile (5) Fish ‘ans. (6) Fish Explanation: Except ish, all others are reptiles. Iustration: Find the odd one out. 6,9, 12, 15, 24, 24, 28, 30 (a) 28 2) 30 (3) 24 (2. (5) 12 Ans. (1) 28, Explanation: Except 28, all others are multiples of 3. Iustration: Choose the pair in which the words are differently related from the rest. (A) Solid: Liquid (3) Sweet: Sour (6) Coloured: Dut (2) Long : High (2) Long: ign (4) Pointed: Blunt ans. Explanation: Except ‘Long : High’ all words in other pairs are ~antonyms of each other. SMARTTIP ‘While answering the questions of ‘Classification and Odd One Out’, you iy P need to: * Know the meaning of the words common types of cams some 5 Banking us ass ity Exam Goalpost fe et iogica reason 30. Reasoning ity aeisenet | snnecimnct reasoning some 5: | Fe prent relationship 10 of the 2, coe Fatigue fatigue cn you might at be are of Foe rove result into Fata if stent ut > meee Sn ead soe Fa iets nto PUNE word ad ane coasting word Yu Fastest Seeman | sre — rowing the mear Pur id ecb eer | a Le ent wiiniaia cael poleschaioehepsere rakes 5 nee eT Simi gn eas Ss cxact meaning > Mevoe without knowing its a pee wnale WOE LESH Fuca er fee 7 > Powis body chour expert Hoot Horse ‘Scan this QR code to wate! art of Horse: speak on oddone out > Hoots a body pas 4. Object: Action int Pravin it ANALOGY _ > Amsco J pang vo o more tems wo show sinlartiesbebvee ‘ni cee thay ee Srcled tare ow tng Aude delvers judgement folios based on anaes cakes raegcaming For sire upose YI sinkera'P nome apr ee 5. Synonyms int on gees? Reorere ov sone Sen rootarear na 00: Ths anloges! eosonng none 7 and apt EEGn Wo ene. K devine ae > and apes Rin one or more aimee Cheer Happy a soars ge Sd’ iy wk snes an > _Hopoys the synonym of chert ananatY ete ets a parla ema |g ieee 4 ateratver trac Setesh to be forge ee mnie a Fesamit reptt. Fr Spat 2 eit Deis Got wnio gp carcass Be nape BS > Ara remtraterstcs of beg cea Ca oveop a snr 2 non-believer $ oF gash fr Best t wou nie oa ee Aogan: aie / o certs ome 8 Pessimis’ relates (0 the > Humble theantnym of arogant haracteristis oba reac ‘£n20al reasoning constites two pes of reasoning: a ahs ate I's atase om crew starts > Allon has, mann, SO! statement o¢ moore ‘and studies the Ie Sant Mies shan inductive reasoning: tt makes {08d generalisations. from Snake: Repu ‘mammal class Sinemet ty fetes on Poros, a te atcenane 2 true ina statement weet ln "the re rove Ree oer neers heer, al Worker co ase, flowers have thom= ‘The “oneusion does not fotow tng ice: Statements ogeany, Poles Police sta, ‘Scanned by CamScanner Sm yoursmahboob. wordpress. COM. s csisaen 140, Spatial eparation nar Pradesh: Lucknow > Lucknow s the captal of Utar Pradesh, samachol Pradesh: Shima » Shima s the capital of Himachal Pradesh Some more examoles Of ansogical reasoning are ghen as follows: iuatration: Bird is related to sky in the same wey 85 Fish is related to (2) water (2) shy (3) food (4) aie (5) none of these Ans. (1) Water Explanation: As bird fhes In the sky. ih lves in the water Therefore, fish 6 related to water, mustration: Clock: Time : Thermometer :? (1) Heat (2) Radiation (3) Eneray (4) Temperature (5) Power ns. (4) Temperature Explanation: As clock 1s used to check time, thermometer is Used to measure temperature. hile answering the quetons of ‘Cssfcation and Oud One Out, you nee to define the relationship beeen Z {0 keywords Look at the snewer options ony when you ae clear bout the relationship: otherwise. you might ‘get confused. Once, you Identify the ‘lationship, search the similar option that can help you In forming a ‘meaningful connection between the ext two words. WORD FORMATION In these type of questions, you need to form a new meaningful Word(s) from the given set of alphabets or words. This type of {Question wil be asked to judge the vocabulary. For example, 1 How many meaningful words can be constituted by using the ESAL letters (using each letter once In a word)? (2) One (2) Two (3) Three () More than Three (5) None of these Ans. (2) By ving he siphabes ES Rand L, we con form two Imsninghl words: Sal ard Sea 2 Wow many meaning! words can be consttted by ting theft fourth, soventh, a nyt nthe word PERMANENT? Gone (3 tee (8 More an Tree (5) None of these Ans. (1) er [MAN N 2 30 laxJsm 6m a) om Wore: Thus, we have formed one meaningful word: TEMP (2) Two Position: Cae SS 7 ‘Anawor the following questions by selecting the most appropriate option. Difficulty Level: LOW Directions (Q. 1-5}: In each of the following questions, five words have been given out of which four are allke In some ‘manner while the fifth one Is different. Choose the odd one out. 5. (2) Mumbai (2) Kanpur (3) Itanagar (4) Dispur (5) Patna Directions (Q. 6-8): Following questions consist of a pair of words bearing a relationship among these, in some or the 1 (a) Noe vera 2fioom ‘other manner, from among the altematives, pick up the pair ge (dh Acrnaeiathe that best ilustrates a similar relationship. (5) Mango 6. Birds: Feathers: Fish: ? 2 (Q Skin (2) Fur (2) Hyundai (2) Honda (3) Scales (4) Shen (3) Volkswagen (4) Tata (5) Flippers gen : (5) Suzuki 7. Dengue: Mosquito: : Plague: ? (A) Dog (2) cat (4) Aero plane (2) Spacecraft 3) Rat (4) Lion (3) Chopper (4) Ship (5) Zebra (5) Train os 8. Calcium: Rickets :: lodine:? (2) Scurvy (2) Goitre (2 ar (2) Motorcycle (3) Growth retardation (4) Anemia Soe 5 (4) Train (8) Xerophthaimia ane — 61 —_ ‘Scanned by CamScanner erin, whlch ferent om th ug easoring Abit Exam Gosipost for Banking Exams mations 20, Choose the cote rection (0 9-3) ach ward ofthe folowing evens (a) 38 - 63 (3 aaa | mint of apa of wore boning» rlaonsih among Osos | ‘Sam ame senate SP O33 ot om ( tet strates a sila lationship. ferent from the 9. Bangles: Hands 21. which ofthe following i 4IMeP¥E Tom the othe | (a Ait: Feet (cane ore ol (3) Orange | (3) L9F (4) Desktop: _ (canner ° (5) vot 5 10. Humans: Lungs: 22, Cup: Crockery Pen (2) Books (1) anole Neve (2) Fan: cis (a) Paper Oe (3) Book: Paper (4) Telephone: Voice (3) Stationery \ {5 Foot Stomach (5) Nib 11 Bulb: Filament 23. whieh of the following has the same relations, (8) Screw Over: Serew (2) Wood: Fre ROCK ROK? 4 (3) Candie: Wick (4) Pen: Pencil (4) BALE: ELAB (2) COLD: Dove (8) Newspaper: Print (3) MEAN: AEMIN (4) MIND: DINM ‘ecton (12-13): Each ofthe folowing questions has (5) CHIN: ICHN Find cut which one ofthe gen alternatives wil be 24, ‘DA’ Isolated to IF, ‘NK willbe related to another member ef the gro ofthat clas (se @)Ps 22, Sea sho, sand, wavee Ger (4) SR (2 wer (2 cont on (5) Boats 1 ies: (25. If 'FI'Is related to "KN" ee al be palate fo 1 aw (2) Ux & renee Tomato wz (a) ur = (2) Nuclear Explosion (5) None of the above (3) Radolgcal emergency (3) Cyber Rae valance 26. Choose the word which Is different from the rest. 44, How many meaning! words (ow (2) Eagle eT RY Meena words can be formed by wing (3) Em (4) Ostricn ‘once ina wont? ° h when one letter is used (5) Peacock (Four ; 190 27. choose the word which Is Drews (4) Six (2) Curd ifterent from the rest, (8) None of hese wee (2) Butter 15. How many meani (5) Cream (4) Cheese fhe Ree ee an be formed by sing 29 i 7 xcoinawory™ MAO when on tris aed 7% Ute itr in tho word "POLITY" ara ne alphabetical order, then how veed in 15) ner (a Pee one nenvietee real th 10 twee () Three 2) Two ‘many meaningful wor e (4) Four any ao on {ng ach eter only once n each wordy, SAMPLE ing the oeeatEhAl words can be Gime 2) Two worgy?” SRAT letters (using each yecorstiuted by 13) Non orteee (8) More tan Tree (8) One letter once In a 3) Th 17. Choose the number {3) Three (2) Two igoeeee Wie nar tome at (5) None o these (4) More than Three (3) 148 30. How | &) 99 63 win a2 tr meaning “SELrIgy St 28e0ng, words 18. Which of | ‘SELFISH’, usi thied, an ‘can be formed, (a) ar he following is trent tom the othery (1) 0 IME 82h letter ong SE lettor of the Won | au (2% (3) Three Y once in each | (3119 ap au (2) Two wort? | IN ofthese (4) Mote than 1 19. Select the Four of This: | (a) Payng wo ana oo ng te (3) Waikir 2) Eating long to ther « &'up. Are alk (5) wring (4) Steping (2) Rey "° hat group” Wh is the oo.” 2 certain (3) Hor that doos 5) Mo 2) wsy 4) Uy Scanned by CamScanner \ Chapter 3: Classification i ines asin. on ao a eee te ete (3) Forest (8) Boox (3) Baryan (8) Deodar Si reamer {8} Sonce 22, omithologit: Bnd: Archacologset:? 4. home ewan ich tae rom he rat (Diisonee 2) Mediators un eer (3) konocoeey (3) Aavave Foe ® amare (8) Deer 24 Peacock nds? 48, choose the word which la tfernt from the ret. (iy mostla (2) America Geom a son &3) missin (@) neane {3} Saurn {@) Mercury {5 Pion rue {35 Carbon: Diamond = Corundum 2 Chan twat whi tere am there (i comet ©) Ruby ) anger > {3 com 8) Tope (3) Humorous (3) Kingness ts) Peer (8) oy 136, NATION : ANTINO : HUNGRY: 2 50, choose the word which ie tferent from the rest. i) HNUGRY @ unnoye Gy rannarbor (2) Sunderban 3) yawoun &@ owner (3) Karronge {@) smcorbet (5) RYGHUN {5} Gangox 37, Safe: Secure: Protect: 7 1. Choose the of one out ia) took @ sue a) 000 @) Fito cua {8} Conserve (3) Conv (a UK (5) saw {5 Ruma 38, BEGK : ADF: PSV¥:7 152. Choose the odd one out &) ROUX (2) onux tse 2) mao taut &&) toor RED (@) QED za ©) weD 39. CORDEN: 2R008Q:: 7: PxIVRO 153. choose the word whch ls terent from the rest. ( MULMUL (2) Susu. (@) crust (2) wartie (3) MUNMUN (@) SASPQL (3) core (4) Lithosphere {8) None ofthe above (5) Panges 40, COMPUTER: FQPRXVHT : LANGUAGE: 2 ‘54. Choose the word which is ifferent from the rest. (a) oxeDIG (2) ocacvie () Spractes (2) Gils (3) ocgncie (4) ocanxcie (3) Lungs (8) Stomata (5) None ofthe above (8) Trachea 41. Choose the word which Is different from the rest. 55, Choose the word whichis ferent from the rest. (a) Teo (2) Sugarcane (a) Possey (2) Haldgnat (3) Rubber (a) Rice (3) Panipat (4) Samat (5) Chak (8) Kurukshetra 42. Choose the word which i diferent from the rest. 56, Nightingale: Warble:: Frog: ? (i) Airport (2) Platform (Yelp (2) Croak (3) Dock (4) Park 2) cache (4) Squeak (8) Bus stand (5) Fone ofthe above 43. Choose the word which Is ffrent from the rest. 57. Four of the following five options are alike In a certain (Ta (2) Huge way and so form a group. Which Is the one that docs (3) Thin (4) Sharp not belong to that group? (5) Smal () sik (2) Nylon i aaa re ern 2 Me (Coon (a) Raa (2) attack (3) Assaur (4) Defence 58. Four ofthe following five options are allke n a certain (5) Ambush way and so form a group. Which ls the one that does 45. Choose the word which is different from the rest. rot belong to that group? (4) Tomato (2) Carrot (2) Bat (2) Spider (8) Ginger (a) Potato (3) Mosquito (4) Buterty 6) eee (8) Cockroach — 63 Scanned by CamScanner reasoning Abily Exam Goalpost fr Banking Exams 158, Choose the word which a diferent from the rest 60. Wom many meaningul words can be formed by using, {he OTRAC letters (ning each letter once in a word)? Tose 2) 490 3) Iwee (4) More than Three 61. How many meaninghul words can be formed by using ‘he AUCSLA letters (using each letter once Ina word)? 3) Tree 4) More than Three ve ay 4. How many tour ieter meaningtu ning! words can be formed, Disrmecnit MR. elghth, and tenth eter ofthe Word wore UTE: walng each letter only once In each None of these vougr WMH ot the following does not (BPS Cierk Mains, 2015) 3) Leat 5) Frat 64 Scanned by CamScanner tort mete scrsineer 2 ioe ne Serer way a 70. Four ofthe fol ‘the one that does nota" se form a pour Wien (S81 PO Mana sont soe (2) Cock (1 Tonoge (4) Crocodie (5) Fog : MODERATE Difficulty Level: MOD! 7 Director (Q 3-5): neoch of he following aveaton, yy “a nave been given out of 1 are words Mavate te fifth one is diferent. Choose the od og oe 2 (2) Karnataka (1 Assam (3) moe (a) sii (3) te 2 (1) 28 (2) 50 (3) 49 (4) 63 (ase 3 (4) tanagp (2) Ganchinagar 3) ima (4) Shitong (3) Duper ‘ 1) Botany (2) Zoology 2 Naty (8) Bochemisy 5 rr 2) 118 29, (3) 78, Direction (@. 6-9: Folowing oor brag questions sine manner, om among oemane 88, It some or oe ‘at best ilustrates a similar relationest PICK UD the pair 2) seen (4) Deck 7. Cheetah cu : 2 Cr CHP Coron Rac 2) Nam lana (4) Foal 8. rt: Feather: Fa 1 es '3) Seales (2) Fur Seo, (8 Sha 9 Language: Lin ust: Dletion aha a sca Toa fn =~ TAQy . * (3) Lexcosepner (4) Hereve stone sovao €9 50-20 Eth sat ott Stent queens ot Stra mn te Sree Snes ee anor ona vo, Exaaanctaet stem ine rroa RR areca nen * a oat Fost (2) River: Stream i mcorNinze (fe Somam (5) Employee: Employer ~ od 2 qr ane 13 Foot Peders (2) Arpine: Arp », (3) partomran (3 owglne: pen My B soe inca WN 42, rectangle: “ (A) Square: Side (2) Triangle: Rectangle G3) Gree Tronge a) ner Se 6) crt: Oa 13. Woodeuttor: Axe (4) Blacksmith: Anvil (3) Architect: Buildings (5) Barracks: Soldier 414, Athlete: Running Track (4) Doctor: Hospital (2) Marketer: Research (3) Teacher: Student (4) Operating System: Windows (6) Letter: Word Direction (Q. 15-19): Each of the following questions has a ‘group. Find out which one of the given alternatives will be ‘another member of the group of that class. 15, Neurology, Cardiology, Pathology (2) Meson: Mansion (4) Plough: Farmer yoursmahboob. wordpress. come s:casicaton 21. How many three letter meaningful words can be formed with ‘the fret, third, and. Mfth lettor of the word "GRAPES: using each lotier only once In each word? (1) One 2) Two (3) Tree {) More than Thee (6) None of these 22, Words are formed with the first, sith, seventh and ninth letters of the word “INTRUDERS” If only one ‘meaningful word Is possible then, which of the following will be the second letter of that word? (it no such word can be formed, gve ‘x’ a8 the answer ‘and if more than one such word can be formed, glve ‘asthe answer.) ae Qt Qe ay x 23, If It Is possibie to make a meaningful word with the fourth, sixth, seventh, elghth, and tenth letter of the word ‘PRESENTATION’, which of the following will be ‘the third letter of that word? (if-no such word can be made, give ‘N’ as the answer and If more than one such word can be made, give” asthe answer.) aR Qt Ge ay ON 24, How many four lettor meaningful words can be formed, with the second, fourth, fifth and seventh letters of th ‘word ‘TRENDSETTER’, using each letter only once in ‘each word? (2) One @) Three () None of these 25. Choose the number, which Is different from the rest. (2) two (4) Four (3 Racy {8 zoey he ? 00 (2) 464 (5) Chemistry = (3) 7353 ‘a 6216 (5) 5125 , 16. Judo, Karate, Wrestling (4) Swimming (2) Basedat 26. Which ofthe following is different from the others? (3) Bosketbal (4) Boxing ay on (2) Sze (5) Disc throw @ au (@) XH 17. Telegram, Postcards, inland letters RUNS (1) Email (2) Post office 27. Select the odd one out. (8) Postage Stamps (4) Computers (a) 3 2) 31 (5) Debentures (3) 77 (4) 19 6) 23 118. Jaipur, Gandhinagar, Mumbal (4) Chennai (2) Jodhpur 28. Which of the following is diferent f kee nw. ‘wnecrmectent in ava (4) Many 28 inns ins thane (5) Naseberry ° (4) Mercury (2) Chlorine 29. Which of the following Is ‘different from thé (3 caton (4) Phosphorus (ae 32 @ukeesr in = (4) DONE = 37 (5) PAN = 20. How many four letter meaningful words can be formed EL PANS St ‘ein the third fifth, sith and ninth letter of the word 30. BLOCKED : YOLXPYW =: 2: OZFMXS sf IATELESS', using each letter only once In each word? (1) DEBATE, (2) RESULT 2 ae (2) Two (3) LABOUR (4) LAUNCH _ (4) Four (8) COLOUR (5) None of these — ees Scanned by CamScanner pesorng ity E10 Gonpost for an agnor : MDELY: HVCONK * Groce mM rence oa (8) hone of above 226: eae ae (3) 31 15 30 manss3et4sF#R:? * pies anar (3) 6= 38 (a) 739 Bans 4 s6:66232:7 8 0 (2 112 ae (4) 128 3 140 ass (aaa (2) 80 aa (2) Te: Shirt (5) Button: Pas (4) Coat: Pocket 38. Latter: Word? (a) Page: Book (2) Prost (3) Cub: Peonie (0 tes ee os Gimeno 38. “Ophthaimia’ Is SESE a sn ier” (2) nose (5) heart (4) bone 40. Choose the aie mrt nlc torent rom the est 8) Cholera (2) Smet pox (9) Tetanus (4) Typhoa 4. Choose the SLE Yd eh a tet te Ge oat (2) Methane” 8 (5) Dieser aeete 42. Choose the trae "Mc tern rom ane (2) Nepnem erst 48, Chey (8 Coin the road ta (ure:Sen {0° words ate aitoren neat Tae (2) Face : tly ‘None of the ap feeb dove, nt: Flower Scanned by CamScanner = a, fa) Ga (2 Ronee above se, Roading i lated t0 KOwtedES It samy Mon isolated to my Gy money (2) jd 3) employment (3) experience {5) engagement 335:216: 997:7 e808 (2) 867 a7 (a) 888 (5) 862 47, Sound : Mufled =? (BMosture:tuma — (2) Colour: Dut (G) despa: Anger (4) Odour: Pungene {5 Noise: Racket 48, SHOE: Nou: REWA:? (@ wor @) ony (3) cr (a) Marv (5) None of he above Bud: Flower: ? (2) Gay: Mud (2) Sapling : {3 mee Gloer {@ Sentng: Tee (5) Paper: Book e 50. Video player: Cassette :: Computer : 2 ) Rees ~ (3) Files. 2) Recordings (5) CPU (4) Floppy 54. “ERID' Is related to“ om ‘DIRE’ in the related to same way as (1) EPIR RIPE ig) (3) EPRI (2) PERI (©) PRE (4) PER (2) Toga (3) KFHI (5) QLNO 83, Lox ) Ri ete ‘54, (0) xe (3) vxsu (8) Moik 55. Four of the mrad 80 fon 2) Triumph (9) Success (8) Grown (6) Wetory 56. Four of the following five options are 2 cortain tray and 20 form 2 group. Which Is the one that does ‘not belong to that group? Gh) Fost (2) Quick. 2 (@) Sudden (4) Ropia (6) Speeay 157. How many four letter meaningful words can be formed tntth the second, fourth, sxth, and seventh leter of the Word ‘STUMBLE’, using each letter only once In each word? (2) One (2) two (a) Tee (4) More than Three (5) None of these 158. Myopla Is related to eyes In the same way a8 pyorthoos 1s rolated to (BPS Clerk Mains, 2045) (€) teeth (2) stomach {@) shoulder (4) tongue (5) chest 59. I I Is possible to make only one meaningful English word with the third, fifth, tenth and twelfth letters of the word REGISTRATION, which would be the second letter of that word from right? If more than one such word can be formed, give 'X' as the answer. If no such word can be formed, give 'Z'as your answer. (IBPS Clerk Prelims, 2015) wz 26 ar an ox 60. Four of the following five are alike in a certain way and. ‘s0 form a group. Which Is the one that does not belong to that group? (SBI PO Mains, 2015) (4) 64 (2) 100 3) 49 (4) 25 6) 36 Difficulty Level: HIGH Directions (Q. 1-5): In each of the following questions, five worls have been given out of which four are alike in some manner while the fifth one Is different. Choose the odd one out. 1 (1) 676 (2) 529 () 784 (4) 841 (6) 852 2 (2) Atal Bihari Vajpayee (2) P. V. Narsimha Rao yoursmahboob. wordpress. Come s.cssstcavon : . pies inet th Bee 3 oy Directions (Q. 6-8): Following ne ot tnords bearing a relationship among these, In ind ‘ther manner, from among. the attematives, ‘that best itustrates 9 lek up the pale ‘similar relationship. : Groves land: ? tr unone (2) Archipelago (3) Broa (6) Steam {8} Coon 1 Quiver Arrows Coven: 7 OMier Ev (3) Pos (3) wenes (8) Corps Sparrows: Host: Crows:? * (rank (2) cate Feet (@) Caraven (5) Murder on (@, 9-20} Each word of the following questions Preto (Og words bearing a rlatonshlp among tomate of 9 pal of Tree son amongst the alteratives:plek wp the pat that best iitustrate a similar relationship. 9. Telephone: Alexander Graham Bell (A) Elevator. Elisha Otis (2) Bulb: Benjamin Franklin (@) Pen: James Watt (4) Steam Engine: John J Loud (6) Computer: Guglielmo Marconi 20. Dendrology: Tress (2) Conchoogy: Crime (3) Agrostology: Grasses () Cetology: ligaments Direction (Q. 14-17): Each of the following questions has a group. Find out which one of the given alternatives will be another member of the group of that class. 11. Cotton, soybean, sugarcane (2) Ecology: Skin (4) Angiology: Humans (4) Wheat (2) Turmeric 3) Peas (4) Barley (6) Mustard 12. Grandeur, Opulence, Splendour (1) Accepted (2) Dejected (3) Intetigent (4) Dry (6) Luxury 13, Ramanujan, Pythagoras, Euclid (3) Lal Bahadur Shashtri (4) Manmohan Sir © Ray Gann 7 nen (2) Gar Friedrich Gauss’ (2) Pablo Picasso (3) Salvador Dali (4) Jane Austen 2 (5) William Shakespeare (1) Binocular (2) Mirror (3) Magnifier (4) Microscope 14. Winnie the Poch, Cincorelia, Mickey Mouse (5) Telescope (2) Donald Duck (2) Flintstones ‘ (3) Tom and Jerry (4) Road Runner (5) Scooby Doo (1) Helium (2) Nitrogen (3) Carbon dioxide (a) Hydrogen 15. Inner core, Outer core, Crust (8) Aluminum (1) Atmosphere (2) Mantle (@) Lang (4) Soi (5) Gasses uu Scanned by CamScanner Reasoning Abilty Eram Goalpost for Banking Exams 16. Neon, Argon, xenon (a) Hetum (3) Nitrogen (5) Carbon Dioxie 417. Cholera, Hepatitis A, Typhold (2) HINA Fis (2) Measles (3) Leptospirosis (4) Pulmonary tuberculosis (6) Cancer 18. {fits possible to make meaningful words withthe first, fourth, seventh, and ninth letter of the word "INCREMENT, which of the following will be the second (2) Metnane (4) Hyarogen letter ofthat word? {if no such word can be formed, give x’ as the answer {and if more than one such word can be formed, give Y" ‘asthe answer) or ar ge ax oy with the 419: Ie posable to make & meaning word st, second, seventh, and elghth letter of the word ‘SEMAPHORE’, which’ of the following will be the s0cond letter ofthat wont? 20. In the following question, 8 roup of letters i (numbered trom 1 to 5) is given combinations of, thee), Nowe’ by five diferent -_ Scanned by CamScanner ianbOo wordpress.com ae sto ea, cambiar oft cri wa garss Brees aa2t sean ot . be mins py Wn 25, lit Ee i comintee fase bert, Stet “te rain ge eet ‘to form a meaningful word. oa arranged Gopi fe fof Ts I» [e ()2,5,14,3 (2) 5,2,4,2,1 Gaasre earas (3.4215 24. In the following question, a group of letters is given (tumbered from 4 to 4} followed by five diferent combinations of these numbers. Select the combination of numbers. so that the letters are ‘arranged accordingly to form a meaningful word. + [2 [3 [a (1)3,4,2,4 menace 34.213 41, 2143 (4)1,2,4,3 25. Choose the m (i357 " MMe" whlch is atferent from the rest, (3) Bato (2) 2356 6) 7261 (4) 6235 26. Which of the (1) RaM=g MME! diferent trom the others? (3) CAT=9 (2) UKE = (5) PAN=3 (4) DONE = 24 27. Select the (2) Water“ On® out Gites (2) Tre (5) Table (4) air 28. Select the (4) ingig ““S °"@ out, (3) North amen spre Bt 29. Select th America (2) Nun “4070 out, (3) Blank (5) Vacant {a} Some ™oty bee 20fF Ee pwections (Q. 20-36% Answer the following questions by sricaing the most appropriate option. 30, choose the odd one out (a or (2) ABA (3 RZ (a) uw (YR ‘34. Choose the odd one out (ay AFD (2) FRHY (aan (4) 1UER (6) KUKR, 132, Choose the odd one out. (a) OAT (2) ces @) RIB 4) TH (6) QUH 33. Choose the odd one out. (a) xw (2) Fo (3) ML (4) PO ots 34, Choose the odd one out. (a) BD ye un (4) PR EF 35. Choose the word which is different from the rest. () Sahara (2) Thar (3) Taklamakan (4) Sunderban (6) Chinuahuan 36. Choose the word which is different from the rest. (2) Potassium (2) Silicon (@) Zirconium (4) Gatium () Germanium 37, How many four letter meaningful words can be formed, with the first, third, seventh, and eleventh letter of the Word ‘TEMPERATURE’, using each letter only once in each word? (4) One (2) Two (3) Three (4) More than Three (6) None of these 38, If it Is possible to make meaningful words with the second, fourth, fifth, eleventh, and thirteenth letter of the Word “ESTABLISHMENT, which of the following will be the fourth letter of that word? if no such word can be made, give ‘N’ as the answer and if more than one such word can be made, give ‘Y’ as the answer. ae (2)T BE ay GN 39. In the following question, a group of letters Is given which are numbered from 1 to §. Below are given five different combinations of these numbers. Which of the {following combination arranged the letters accordingly to form a meaningful word. yoursmahboob.wordpress.cOM, ., csnsin (a) TeeHoPsy (a) TICHCOPSY {6} None of the stove ‘42. Choose the odd one out. a) 80 (2) HK (3) MP (a) NO ow 142, Choose the odd one out. a) HSA (2) MUN (@) OLKP, (4) PIX (5) WOOK 43, ZRYQ: KCB : PWOV: 7 (a) GBHA Qs (3) ELK (@) EOF (6) None of the above meaningful word with the 44, 11 It Is possible to make meant mn te Second, agin, steventh 2nd Freer eee era, wich of the following vl be Bete eter ofthat wore? fo such word can tne a nas the anewer ad if more than one such Mord can bemade, give as the answer? aoe Qt @E ay fan It ts possible to make meaningful word with the first, second, fifth, and sixth letter of the Word ‘DANGER’, ‘which of the following will be the fourth letter of that word? If no such word can be made, give 'N' as the ‘answer and if more than one such word can be made, give Y's the answer? 45. (@R yt @eE ay ON 46. How many four letter meaningful words can be formed, ‘with the first, third, fifth, and seventh letter of the Word ‘POSITION’, using each letter only once in each word? (2) One (2) Two (3) Three (4) More than Three (6) None of these 47. In the following question, a group of letters is given which are numbered from 1 to 4, Below are given five different combinations of these numbers. Which of the following combination arrange the letters accordingly to form a meaningful word? EPHO (1) 4,4,3,2 2) 2.4.4.3 (3) 4,3.1,2 434.42 6) 3,4,.2,4 In the following question, a group of letters is given which are numbered from 1 to 5. Below are given five different combinations of these numbers. Which of the following combination arrange the letters accordingly, to form a meaningful word? HRCIA AGSUR (1)2,1,4,5,3 (2) 2,4,5,3,4 a )3,4,1,2,5 (2)3.4,2.1.5 3.4542 43.1524 se (5) 3,4.5,1,2 ane (4) 4,3.5,1,2 40. NEUROTIC: TICRONEU :: PSYCHOTIC :? (4) TicconPsY (2) TICOCHPSY —_ Scanned by CamScanner “Rewolag Abily Em Gonpost for Banking Exams © un i! sermerabetomete on ane mening word tom $0. How many a seventy tats of the. word newer AT th the tid Veto om ht Your word? sor at ib tment (i) Neve formed es Mee ha one aach word tat be (3) Two greeranwer ars em an, 201) (8) None ofthese ay ax 70 ‘Scanned by CamScanner ee uccad = 3s_| 5 2 aT a a aes. 39) 6 fa a5_| a tat he ze at 2 ale 2 | @ a7 | @ za | 3 [2 2 Le 2 | @ as | ae | 48) af 20 @ a | @ ae | 3] 2 “a Tw nu | so |) a [ o 6 | @ 45 Tw 2 | @ sx | w ze {| 7] «| 3 2 | @ 2 | 2 2 | @ a | @ a7 {6 a | 33 | 4) 30_}_@) sha | @ 3 | ® | @ aa 1) 30] 2 | 8) as | | 32 | pao so | 6) a | so | 33 | 2] sa w as [| a s7_| «| © Ce 32 |) 29 | se a | 4] ss [9 20 | ay so | @ 36 | @ 3 | sa Ta) a | @ eo | a7 | ato 5 | @ zw a | | ra so | 2 23 | Ques. | Ans. 3 | a ie 7 | 2 2 [2 1 | «| 3 3) so] 2 | 8) 2 [a a | =o |e so 26 | a) 3 [a a2 [ 2] Com a | @ 2 |e a3 | 2 |) a fw 2 | @ s | @ “| @ 2 18 e2 | aw 2 | @ 6 | @ a [ aa 2 | aw 30 | a 7 | a | 25] 2) eo | a a s | a7 | © 26 | (2) S| s2_| 9 a 438 | 27 |) 66 | 4) 2 fe 2 | @ a2 | ©) 2] 2 a | @ «4 |e a [a so | @ 23] es | a) 3 | © 2 |e 30 | a) eo | @ Ee) 3B | a) 70 | a [| @ wu [Tw ta Co pa =e 3 1) 3 [@ 1s | Ee) 1] @ 40 | 2 v | @ 35 | 2) 2 [2 a1 | 3 [6 3% | 2 3 [2 a | 3 vw |e L37_| @ 4 (5) 43 3 20 (6) —_ Scanned by CamScanner Meigen re a, ben rg. ae ton ote * Monotie "compares ear tm travel on (2) Alothers are used to the earth whereas Spacecrat used to travel in space. ‘4 (8)Al atvers move on land, except ‘ero plane, ‘5 (2) A others ae capitals of indian States, except Kanpur. 6. (3) Bids have feathers os skin whereas fish have scales as shin, 7%) Dengue i caused by asauiioes whereas plague is caused by rats & (2) Riches isa disease caused by the ceficency of caloum whereas efisiency of Todine results in gotve. % (4) As bangles are wom in hands, ‘anklets are worn in feet, 30 (2) As humans respke though !unes. fish respie through git 44 (3) 4s foment is the part of bub, Wick s the part ofa cance, 42 (3) Sea shets, sand and waves are related to see, Calamity that occurs when snow rapidly towards his, (9) By using the letters T,P,$ and ©. we can s 45. (4) By using the letters ‘E51 9° ond 1. we can farm fan Reena words: USER, RuES, RUSE, SUER and SURE. 48 © The letter which are at the fourth, and sixtn positions Elie Word SAMPLE” are AP ane fF omectily By using the lees wo 2nd E, we can form tne Words: APE and Pes, ‘Scanned by CamScanner Perinnas vd ony 37 ale options Sone nb Enesen tosing_ the 3 pate ltr pston, sind ters om te at eter ‘oer seamen ao whe At tas so 8 cronies of 8 peso, erent sleeping. 20. (5)Allothers are written in reverse order, except 35 ~ 51. * Ccouianéerepimeycani “oy under the % aes et “erion si, ‘pen comes under the category of ‘Saionery. 23. (5)hs, 1234 3124 Rock:c Rok Therefore, A2BA 8 bal CHIN: + oHN 24 (aya, Dk 195s toe Simiary, Nok 15s sop 25. (2)As, Foy 151s Koow Simitary, Pos 1515. 29. (1) By using thé alphabets, ‘and T, we can form 6 meaningful word: ROAST, 30. (1) Word: Position: Tre We Cn om ony on LESS. 34 (4) Except 4) in all options th 's @ certain gap between tates @. ~ 482. In option (1) gapig29| 40. 32 (4) "Man uses ‘Up’ 10 di! ¢ Similarly, ‘beak’ i used by te “bir to pick things. 5S An omhoog studies ato, ‘bids’. Similarly, ‘afchaeologist’is involved with ee study of ‘archaeology. fi 54 (3) ‘Peacock’ is the national big ve of india. Similar, “Emu is the National bird of Australia, a 38 2) As ‘ciamonat is made out of ‘Saron’ ‘ruby’ is: made cat of ‘corundum’ 2 36. (2) A, 223456 465 2 a NATi on: No Therefore, 123 465 Hun YR ANT4 * Bie 2 4 SRY uawg 37, (2) Sate’ has the sa se sei St game . ure og ether

Potrebbero piacerti anche